Pediatrics Exam 2

Pataasin ang iyong marka sa homework at exams ngayon gamit ang Quizwiz!

Which is the most critical element of pediatric emergency care? *a.* Airway management *b.* Prevention of neurologic impairment *c.* Maintaining adequate circulation *d.* Supporting the child's family

*a.* Airway management (Airway management is the most critical element in pediatric emergency care. Prevention of neurologic impairment is certainly a concern in pediatric emergency care; however, it is not considered the most critical element. Maintaining adequate circulation is accomplished after a patent airway is established. The focus of emergency care is stabilizing the child's physiologic status. Supporting the family is important, but it is not considered to be the most critical element in pediatric emergency care.)

What should be the emergency department nurse's next action when a 6-year-old child has a systolic blood pressure of 58 mm Hg? *a.* Alert the physician about the systolic blood pressure. *b.* Comfort the child and assess respiratory rate. *c.* Assess the child's responsiveness to the environment. *d.* Alert the physician that the child may need intravenous fluids.

*a.* Alert the physician about the systolic blood pressure. (Hypotension is a late sign of shock in children. The lower limit for systolic blood pressure for a child more than 1 year old is 70 mm Hg plus two times the child's age in years. A systolic blood pressure of 58 mm Hg calls for immediate action. The nurse should be direct in relaying the child's condition to the physician. This action does not address the problem of shock, which requires immediate intervention. Assessing the child's responsiveness is included in a neurologic assessment. It does not address the systolic blood pressure of 58 mm Hg. Although this child most likely requires intravenous fluids, the physician must be apprised of the systolic blood pressure so that appropriate intervention can be initiated.)

An emergency department nurse is making a general appearance assessment on a preschool child just admitted to the emergency department. Which general assessment findings indicate the child "looks bad"? *Select all that apply.* *a.* Color pale *b.* Capillary refill less than 2 seconds *c.* Unwilling to separate from parents *d.* Cold extremities *e.* Lethargic

*a.* Color pale *d.* Cold extremities *e.* Lethargic (Signs of a child "looking bad" on a general appearance assessment include pale skin, cold extremities, and lethargy. A capillary refill of less than 2 seconds is a "good sign" as well as a child who is unwilling to separate from parents [separation anxiety, expected].)

You are the nurse working triage in the emergency department. A school-age child is brought in for treatment, carried by her mother. What is part of a primary assessment that should be performed first on this child? *a.* Determine level of consciousness. *b.* Obtain a health history. *c.* Obtain a full set of vital signs. *d.* Evaluate for pain.

*a.* Determine level of consciousness. (A primary assessment consists of assessing the child's airway, breathing, circulation, level of consciousness, and exposure (ABCDEs). Obtaining the child's health history is a component of a secondary assessment. Vital signs are included in a secondary assessment, after the ABCDEs are assessed. Assessing for pain is a component of a secondary assessment.)

What is the goal of the initial intervention for a child in cardiopulmonary arrest? *a.* Establishing a patent airway *b.* Determining a pulse rate *c.* Removing clothing *d.* Reassuring the parents

*a.* Establishing a patent airway (The first intervention for a child in cardiopulmonary arrest, as for an adult, is to establish a patent airway. Assessment of pulse follows establishment of a patent airway. Clothing may be removed from the upper body for chest compressions after a patent airway is established. The first priority is to establish a patent airway.)

A 2-year-old child is in the playroom. The nurse observes him picking up a small toy and putting it in his mouth. The child begins to choke. He is unable to speak. Which intervention is appropriate? *a.* Heimlich maneuver *b.* Abdominal thrusts *c.* Five back blows *d.* Five chest thrusts

*a.* Heimlich maneuver (To clear a foreign body from the airway, the American Heart Association recommends the Heimlich maneuver for a conscious child older than 1 year of age. Abdominal thrusts are indicated when the child is unconscious. Back blows are indicated for an infant with an obstructed airway. Chest thrusts follow back blows for the infant with an obstructed airway.)

What is the nurse's immediate action when a child comes to the emergency department with sweating, chills, and fang bite marks on the thigh? *a.* Secure antivenin therapy. *b.* Apply a tourniquet to the leg. *c.* Ambulate the child. *d.* Reassure the child and parent.

*a.* Secure antivenin therapy. (Antivenin therapy is essential to the child's survival because the child is showing signs of envenomation. The use of a tourniquet is no longer recommended. When a bite or envenomation is located on an extremity, the extremity should be immobilized. Envenomation is a potentially life-threatening condition. False reassurance is not helpful for building a trusting relationship.)

The emergency department nurse notices that the mother of a young child is making a lot of phone calls and getting advice from her friends about what she should do. This behavior is an indication of *a.* Stress *b.* Healthy coping skills *c.* Attention-getting behaviors *d.* Low self-esteem

*a.* Stress (Hyperactive behavior such as making a lot of phone calls and enlisting everyone's opinions is a sign of stress. The behavior described is not a healthy coping skill. This may be an attention-getting behavior but is more likely an indicator of stress. This mother may have low self-esteem, but the immediate provocation is stress.)

Which action should the nurse working in the emergency department implement in order to decrease fear in a 2-year-old child? *a.* Keep the child physically restrained during nursing care. *b.* Allow the child to hold a favorite toy or blanket. *c.* Direct the parents to remain outside the treatment room. *d.* Let the child decide whether to sit up or lie down for procedures.

*b.* Allow the child to hold a favorite toy or blanket. (It may be necessary to restrain the toddler for some nursing care or procedures. Because toddlers need autonomy and do not respond well to restrictions, the nurse should remove any restriction or restraint as soon as safety permits. Allowing a child to hold a favorite toy or blanket is comforting. Parents should remain with the child as much as possible to calm and reassure her. The toddler should not be given the overwhelming choice of deciding which position she prefers.)

What may cause hypovolemic shock in children? *Select all that apply.* *a.* Hyperthermia *b.* Burns *c.* Vomiting or diarrhea *d.* Hemorrhage *e.* Skin abscess that cultures positive for methicillin-resistant Staphylococcus aureus (MRSA)

*b.* Burns *c.* Vomiting or diarrhea *d.* Hemorrhage (These are all causes of hypovolemic shock, which is characterized by an overall decrease in circulating blood or fluid volumes.)

You are the nurse caring for a child who is diagnosed with septic shock. He begins to develop a dysrhythmia and hemodynamic instability. Endotracheal intubation is necessary. The physician feels that cardiac arrest may soon develop. What drug do you anticipate the physician will order? *a.* Atropine sulfate *b.* Epinephrine *c.* Sodium bicarbonate *d.* Inotropic agents

*b.* Epinephrine (Atropine sulfate is used to treat symptomatic bradycardia. Epinephrine is the drug of choice for the management of cardiac arrest, dysrhythmias, and hemodynamic instability. Sodium bicarbonate is given to treat severe acidosis associated with cardiac arrest. Inotropic agents are indicated for hypotension or poor peripheral circulation in a child.)

Which nursing action facilitates care being provided to a child in an emergency situation? *a.* Encourage the family to remain in the waiting room. *b.* Include parents as partners in providing care for the child. *c.* Always reassure the child and family. *d.* Give explanations using professional terminology.

*b.* Include parents as partners in providing care for the child. (Allowing the parents to remain with the child may help calm the child. Include parents as partners in the child's treatments. Parents may need direct guidance in concrete terms to help distract the child. Telling the truth is the most important thing. False reassurance does not facilitate a trusting relationship. Professional terminology may not be understood. Speak to the child and family in language that they will understand.)

How should the nurse instruct the mother who calls the emergency department because her 9-year-old child has just fallen on his face and one of his front teeth fell out? *a.* Put the tooth back in the child's mouth and call the dentist right away. *b.* Place the tooth in milk or water and go directly to the emergency department. *c.* Gently place the tooth in a plastic zippered bag until she makes a dental appointment. *d.* Clean the tooth and call the dentist for an immediate appointment.

*b.* Place the tooth in milk or water and go directly to the emergency department. (The parent may replace the tooth incorrectly, so it is best not to advise the parent to do this. The parent should be told to keep the tooth moist by placing it in a saline solution, water, milk, or a commercial tooth-preserving solution and get the child evaluated as soon as possible. The tooth should be kept moist, not dry. The child should be evaluated as soon as possible. Cleaning or scrubbing the tooth could damage it. It is essential for the child to have an immediate dental evaluation.)

Assessment of a child with a submersion injury focuses on which system? *a.* Cardiovascular *b.* Respiratory *c.* Neurologic *d.* Gastrointestinal

*b.* Respiratory (Cardiovascular assessment is secondary to the airway and breathing. Assessment of the child with a submersion injury focuses on the respiratory system. The airway and breathing are the priorities. Preventing neurologic impairment is a goal of intervention. Because the primary problem in submersion injuries is hypoxia, the focus of assessment is the respiratory system. Gastrointestinal assessment is less of a priority than assessment of other body systems.)

A preschool child in the emergency department has a respiratory rate of 10 breaths per minute. How should the nurse interpret this finding? *a.* The child is relaxed. *b.* Respiratory failure is likely. *c.* This child is in respiratory distress. *d.* The child's condition is improving.

*b.* Respiratory failure is likely. (Although the respiratory rate slows when an individual is relaxed, a rate of 10 breaths per minute in an ill preschool child is not a normal finding and is cause for concern. Very slow breathing in an ill child is an ominous sign, indicating respiratory failure. A rapid respiratory rate indicates respiratory distress. Other signs of respiratory distress may include retractions, grunting, and nasal flaring. A respiratory rate of 10 breaths per minute is not a normal finding for a preschool child. This conclusion is incorrect.)

What condition does the nurse recognize as an early sign of distributive shock? *a.* Hypotension *b.* Skin warm and flushed *c.* Oliguria *d.* Cold, clammy skin

*b.* Skin warm and flushed (Hypotension is a late sign of all types of shock. An early sign of distributive shock is extremities that are warm to the touch. The child with distributive shock may have hypothermia or hyperthermia. Oliguria is a manifestation of hypovolemic shock. Cold, clammy skin is a late sign of septic shock, which is a type of distributive shock.)

Which observations made by an emergency department nurse raises the suspicion that a 3-year-old child has been maltreated? *a.* The parents are extremely calm in the emergency department. *b.* The injury is unusual for a child of that age. *c.* The child does not remember how he got hurt. *d.* The child was doing something unsafe when the injury occurred.

*b.* The injury is unusual for a child of that age. (The nurse should observe the parents' reaction to the child but must keep in mind that people behave very differently depending on culture, ethnicity, experience, and psychological makeup. An injury that is rarely found in children or is inconsistent with the age and condition of the child should raise suspicion of child maltreatment. The child may not remember what happened as a result of the injury itself, for example, sustaining a concussion. Also, a 3-year-old child may not be a reliable historian. The fact that the child was not supervised might be an area for health teaching. The nurse needs to gather more information to determine whether the parents have been negligent in the care of their child.)

The nurse observes abdominal breathing in a 2-year-old child. What does this finding indicate? *a.* Imminent respiratory failure *b.* Hypoxia *c.* Normal respiration *d.* Airway obstruction

*c.* Normal respiration (A very slow respiration rate is an indicator of respiratory failure. Nasal flaring with inspiration and grunting on expiration occurs when hypoxia is present. Young children normally exhibit abdominal breathing. When measuring respiratory rate, the nurse should observe the rise and fall of the abdomen. The child with an airway obstruction will use accessory muscles to breathe.)

Which action should the nurse incorporate into a care plan for a 14-year-old child in the emergency department? *a.* Limit the number of choices to be made by the adolescent. *b.* Insist that parents remain with the adolescent. *c.* Provide clear explanations and encourage questions. *d.* Give rewards for cooperation with procedures.

*c.* Provide clear explanations and encourage questions. (Because adolescents are capable of abstract thinking, they should be allowed to make decisions about their care. Adolescents should have the choice of whether parents remain with them. They are very modest, and this modesty should be respected. Adolescents are capable of abstract thinking and can understand explanations. They should be offered the opportunity to ask questions. Giving rewards such as stickers for cooperation with treatments or procedures is more appropriate for the younger child.)

A child is brought to the emergency department. When he is called to triage, which vital sign should be measured first? *a.* Temperature *b.* Heart rate *c.* Respiratory rate *d.* Blood pressure

*c.* Respiratory rate (Temperature should be measured after other vital signs because it can be upsetting for children. Heart rate is not the first vital sign measured in children. When taking children's vital signs, the nurse observes the respiratory rate first. Blood pressure is taken after other vital signs because it can be upsetting for children.)

What is an appropriate nursing intervention for a 6-month-old infant in the emergency department? *a.* Distract the infant with noise or bright lights. *b.* Avoid warming the infant. *c.* Remove any pacifiers from the baby. *d.* Encourage the parent to hold the infant.

*d.* Encourage the parent to hold the infant. (Distraction with noise or bright lights is most appropriate for a preschool-age child. In an emergency health care facility, it is important to keep infants warm. Infants use pacifiers to comfort themselves; therefore the pacifier should not be taken away. Parents should be encouraged to hold the infant as much as possible while in the emergency department. Having the parent hold the infant may help to calm the child.)

A 3-year-old is brought to the emergency department by ambulance after her body was found submerged in the family pool. The child has altered mental status and shallow respirations. She did not require resuscitative interventions. Which condition should the nurse monitor first in this child? *a.* Neurologic status *b.* Hypothermia *c.* Hypoglycemia *d.* Hypoxia

*d.* Hypoxia (Although a neurologic assessment will be required, it is not the area of primary assessment. The airway is always assessed first. Hypothermia offers protection to the brain. It is a concern, but not the area of primary concern. Although the child may have electrolyte imbalances, this is not the primary assessment area. Hypoxia is responsible for the injury to organ systems during submersion injuries. Hypoxia can progress to cardiopulmonary arrest. Monitoring the airway is always the number one concern.)

What is the leading cause of unintentional death in children younger than 19 years of age in the United States? *a.* Drowning *b.* Airway obstruction *c.* Pedestrian injury *d.* Motor vehicle injuries

*d.* Motor vehicle injuries (Drowning is the second leading cause of unintentional death for children under 19 years of age. Airway obstruction is the third leading cause of unintentional death for children under 19 years of age. Pedestrian injuries are not the leading cause of unintentional death in children. It is a significant problem, with most injuries occurring in children between 1 and 4 years. The Centers for Disease Control and Prevention (CDC) has consistently found that motor vehicle injuries are the leading cause of unintentional death in children younger than 19 years of age in the United States.)

The father of a child in the emergency department is yelling at the physician and nurses. Which action is contraindicated in this situation? *a.* Provide a nondefensive response. *b.* Encourage the father to talk about his feelings. *c.* Speak in simple, short sentences. *d.* Tell the father he must wait in the waiting room.

*d.* Tell the father he must wait in the waiting room. (When dealing with parents who are upset, it is important not to be defensive or attempt to justify anyone's actions. Encouraging the father to talk about his feelings may assist him to acknowledge his emotions and may defuse his angry reaction. People who are upset need to be spoken to with simple words (no longer than five letters) and short sentences (no more than five words). Because a parent who is upset may be aggravated by observers, he should be directed to a quiet area.)

In which situation is the administration of milk or water indicated after ingestion? *a.* The child is suspected of ingesting lead paint chips. *b.* The child ingested approximately 15 tablets of baby aspirin. *c.* The child ingested an over-the-counter product containing acetaminophen. *d.* The child ingested an acid or alkali.

*d.* The child ingested an acid or alkali. (Ingestion of leaded paint chips does not indicate treatment with administration of water or milk. Ingestion of aspirin is not treated with administration of water or milk. The treatment may involve gastric lavage with activated charcoal, IV fluids with various additives to decrease absorption, treatment of electrolyte imbalances, and vitamin K for bleeding tendencies. Ingestion of acetaminophen is not treated with administration of milk or water. Gastric lavage within 1 hour and administration of the antidote N-acetylcysteine (Mucomyst) is indicated. Administering water or milk can dilute the toxic effects of acid or alkali ingestion.)

Which initial assessment made by the triage nurse suggests that a child requires immediate intervention? *a.* The child has thick yellow rhinorrhea. *b.* The child has a frequent nonproductive cough. *c.* The child's oxygen saturation is 95% by pulse oximeter. *d.* The child is grunting.

*d.* The child is grunting. (Nasal discharge indicates that the child has a respiratory condition but does not mean the child needs immediate attention. A productive cough is not a finding that indicates that the child requires immediate attention. An oxygen saturation of 95% is a normal finding. One of the initial observations for triage is respiratory rate and effort. Grunting is a sign of hypoxemia and represents the body's attempt to improve oxygenation by generating positive end-expiratory pressure.)

Which nursing action is most appropriate to assist a preschool-age child in coping with the emergency department experience? *a.* Explain procedures and give the child at least 1 hour to prepare. *b.* Remind the child that she is a big girl. *c.* Avoid the use of bandages. *d.* Use positive terms and avoid terms such as "shot" and "cut."

*d.* Use positive terms and avoid terms such as "shot" and "cut." (Preschool-age children should be told about procedures immediately before they are done. Allowing 1 hour of time to prepare only allows time for fantasies and increased anxiety. Children should not be shamed into cooperation. Bandages are important to preschool-age children. Children in this age-group believe that their insides can leak out and that bandages stop this from happening. Using positive terms and avoiding words that have frightening connotations assist the child in coping.)

Parents ask the nurse for advice when telling their 4-year-old about a grandmother's death. The nurse's best response involves teaching the parents that the child's concept of death is: 1 temporary. 2 permanent. 3 personified in various forms. 4 inevitable at some age.

1 Death is seen as a temporary departure. Death is thought of as not permanent; life and death can change places with each other. Personification is typical of school-age children. Children 9 to 10 years old have this understanding of death.

The nurse finds that the parent of a terminally ill child is unusually optimistic about the child's condition. What should the nurse interpret from this? The parent: 1 Is in the denial stage of grief. 2 Believes the diagnosis is wrong. 3 Normally has a cheerful nature. 4 Is optimistic about the treatment

1 Parents may be in a state of shock when first hearing about the terminal illness of their child. The parent's abnormal optimism about the child's condition indicates that the parent is in denial and has not accepted the child's illness. It is unlikely that the parent believes that the diagnosis is wrong. The diagnosis of a terminally ill condition in a child is depressing for any parent, even one with a cheerful nature. Being optimistic when there is no cure indicates the stage of shock and denial.

A child who was terminally ill died. What intervention should the nurse provide for the child and the family? 1 Allow the family be with and hold or rock the child. 2 Do not remove any of the tubes or catheters. 3 Follow the hospital policy to administer last rites. 4 Instruct the family that the nurse bathes the body.

1 The nurse should be very sensitive to the emotions of the family members who are grieving their loss and try to support them as much as possible. The nurse should allow privacy to the family with the child and allow them to hold or rock the child if they want. The nurse should remove the tubes and catheters from the child's body and let the family hold the child. The nurse should allow the family to provide last rites according to their culture rather than following hospital policy. It helps the family cope. The family should be allowed to bathe and dress the child, as per their cultural practice.

The nurse in the pediatric unit is admitting a 2½-year-old child. Which stage in Erikson's psychosocial stages of development should the nurse plan care around? 1. Trust versus Mistrust 2. Initiative versus Guilt 3. Industry versus Inferiority 4. Autonomy versus Shame and Doubt

4. Autonomy versus Shame and Doubt

Which child behaviors indicate maladaptive coping toward chronic illness? Select all that apply. 1 Is irritable both at home and during school 2 Demonstrates feelings of pessimism and inadequacy 3 Expresses feelings of worthlessness to others 4 Expresses anxiety during remissions 5 Demonstrates confidence during an exacerbation

1, 2, 3 Children with negative and declining attitudes toward their chronic illness use maladaptive coping patterns. These maladaptive coping patterns are characterized by irritability at home and school. The child has a pessimistic outlook toward the disease and feels inadequate in managing it. The child may feel different, withdrawn, and less worthy than others. The child who has healthy coping skills may expresses appropriate emotions, such as anxiety, sadness, and anger, during periods of illness exacerbations. Demonstrating confidence during remissions indicates healthy coping skills

What information should the nurse provide to a group of nursing students when teaching them about the analgesic stepladder for pain management in children with terminal illness? Select all that apply. 1 "It was implemented by the World Health Organization (WHO)." 2 It provides for the use of adjunctive drug therapy to ease pain." 3 It outlines the principles of analgesic selection and titration." 4 "It describes the use of combination therapy to decrease side effects." 5 It was created to prevent drug dependence and drug addiction."

1, 2, 3 Children with terminal illness experience some amount of pain in the last stage of their illness. Nurses should follow the analgesic stepladder framed by World Health Organization (WHO) for pain management in children. It outlines the use of adjunctive therapy depending on the severity of the pain as mild or severe/persistent. It outlines the principles of analgesic selection such as administration of nonnarcotic analgesics to ease mild pain and administration of narcotic analgesics (opioids) to ease severe or persistent pain. It outlines the use of combination therapy to counteract adverse effects, not side effects. A patient with a terminal illness will require increased levels of pain medication to manage pain. The patient will not become dependent or addicted.

Which approaches should the nurse adopt to provide support and guidance to a grieving family? Select all that apply. 1 Follow up regularly with telephone calls. 2 Provide a referral to a support group. 3 Recommend bereavement counseling. 4 Refrain from sending them cards. 5 Ask the family come for an office visit.

1, 2, 3 Nurses play an important role in helping the grieving family cope with their loss. The nurse should plan regular follow-ups through phone calls or meetings to assess whether they are coping well. Support groups and bereavement counseling may help the family vent their feelings and learn coping skills. The nurse should initiate contact by sending cards. The nurse should not expect the family to come in for a visit. The nurse should ask permission to make a home visit to make it easier on the family. This helps the family feel cared for.

The parents of a 12-year-old child are informed that the child is in the terminal stages of osteosarcoma. The nurse explains to the parents about the care that needs to be provided for the child. Which characteristics in the parents indicate that they are in denial? Select all that apply. 1 Want the child to be admitted for more treatment 2 Refuse to believe the results of the tests 3 Pleased to receive the medical diagnosis 4 Ask questions about treatment or prognosis 5 Insist that nobody is telling them the truth

1, 2, 3, 5 The initial diagnosis of a terminal illness in a child is met with shock or denial. Parents experience this reaction, and it can last for days, weeks, or months. If the parents ask questions about the child's treatment or prognosis, this indicates that they have accepted the diagnosis. Parents in denial may want the child to be admitted for more medical treatment. They may feel the terminal diagnosis is wrong and refuse to believe the test results. People are usually not pleased to receive a terminal diagnosis; they are upset and angry. Insisting that nobody is telling them the truth is also an indicator of denial.

Which behaviors of the family members indicate avoidance coping behavior? Select all that apply. 1 Refuses to agree to the treatment 2 Demonstrates anger and hostility 3 Expresses deep feelings of sorrow 4 Makes realistic plans for the future 5 Seeks cures from the faith healers

1, 2, 5 Coping reactions to stressors include those that indicate avoidance. Avoidance includes behaviors that help the person deny the truth or escape from it. Refusing to agree to treatment, being angry toward the staff, and seeking nontraditional medical treatments such as faith healing are examples of avoidance behavior. Expressing feelings of sorrow and planning realistically for the future are approach behaviors that indicate acceptance of the condition.

The parents of a terminally ill child ask the nurse how hospice care is different from hospital care. What information should the nurse give them? Select all that apply. 1 No extraordinary efforts would be made to prolong the child's life. 2 Pain and control of symptoms are the primary concerns of treatment. 3 The child's needs are considered to be the priority over the family's needs. 4 Hospice professionals are solely responsible for providing care to the child. 5 Family members are the principal caregivers supported by the hospice team.

1, 2, 5 Nurses should offer information about hospice care to the parents of a terminally ill child. Hospice is a community health care organization which specializes in caring for a dying child. It combines principles of hospice care and palliative care. The client is taken care of at home in the final stages of life. The concepts that make it different from hospital care include: no extraordinary efforts are made to prolong the child's life, and pain and control of symptoms are the primary concerns of treatment. The family members are usually the principal caregivers supported by a team of professionals, and the family's needs are given the same importance as the child's needs.

A baby is diagnosed with Turner syndrome shortly after birth. The parent is showing signs of depression and is finding it difficult to bond with the baby. Which characteristics indicate that the parent is in the adjustment stage? Select all that apply. 1 Assumes that the disability is genetically inherited 2 Acts happy and optimistic despite the diagnosis 3 Focuses on caregiving as the condition progresses 4 Believes the disability happened during pregnancy 5 Believes the disability is a punishment from God

1, 4, 5 The adjustment stage is the next phase after shock and is most often characterized by an open admission that the condition exists. This stage is accompanied by responses such as guilt. Many parents may feel responsible for the disability because it was genetically inherited. The mother may also believe she caused the disability by doing something wrong during pregnancy or may believe God is punishing her for previous misdeeds. Acting happy and optimistic despite the revealed diagnosis indicates the parent is still in denial and has not accepted the child's disability. Focusing on new caregiving practices as the condition progresses indicates total acceptance of the condition.

The registered nurse (RN) is educating a new RN on the "law and order orientation" found in level 2 of Kohlberg's theory of moral development. Which statement by the new RN indicates that the teaching has been effective? 1. "An example of this is: If I skip down the hall, will the teacher be mad at me?" 2. "An example of this is: We will spend time talking about the activities for the week." 3. "An example of this is: I don't like it when you yell while I am talking to my friend. Here are some activities to do until I am finished talking." 4. "An example of this is: If you do all of your classwork today without bothering others in the class, you will get an extra 'seed' for your good behavior garden."

1. "An example of this is: If I skip down the hall, will the teacher be mad at me?"

The clinic nurse provides information to the mother of a toddler regarding toilet training. Which statement by the mother indicates a need for further information regarding toilet training? 1. "Bladder control usually is achieved before bowel control." 2. "The child should not be forced to sit on the potty for long periods." 3. "The ability of the child to remove clothing is a sign of physical readiness." 4. "The child will not be ready to toilet train until the age of about 18 to 24 months."

1. "Bladder control usually is achieved before bowel control."

A mother of a 4-year-old expresses concern because her hospitalized child has begun thumb sucking. The mother states that this behavior began 2 days after hospital admission. Which response by the nurse is appropriate? 1. "It is best to ignore the behavior." 2. "Your child is acting like a baby." 3. "A 4-year-old is too old for this type of behavior." 4. "The health care provider will need to be notified."

1. "It is best to ignore the behavior."

The mother of a 5-year-old child tells the nurse that the child scolds the floor or a table if she hurts herself on the object. The nurse educates the mother according to Piaget's theory of cognitive development and its terminology and definitions. Which statement by the mother indicates that the teaching has been effective? 1. "This is an example of animism." 2. "This is an example of egocentric speech." 3. "This is an example of object permanence." 4. "This is an example of global organization."

1. "This is an example of animism."

A 2-year-old child is treated in the emergency department for a burn to the chest and abdomen. The child sustained the burn by grabbing a cup of hot coffee that was left on the kitchen counter. The nurse reviews safety principles with the parents before discharge. Which statement by the parents indicates an understanding of measures to provide safety in the home? 1. "We will be sure not to leave hot liquids unattended." 2. "I guess our children need to understand what the word hot means." 3. "We will be sure that the children stay in their rooms when we work in the kitchen." 4. "We will install a safety gate as soon as we get home so the children cannot get into the kitchen."

1. "We will be sure not to leave hot liquids unattended."

A 7-year-old child is hospitalized with a fracture of the femur and is placed in traction. In meeting the psychosocial needs of the child, the nurse most appropriately selects which play activity for the child? 1. A board game 2. A large puzzle 3. A finger-painting set 4. A coloring book with crayons

1. A board game

The mother of a 3-year-old asks a clinic nurse about appropriate and safe toys for the child. The nurse should tell the mother that the most appropriate toy for a 3-year-old is which? 1. A wagon 2. A golf set 3. A farm set 4. A jack set with marbles

1. A wagon

The maternity nurse is providing instructions to a new mother regarding the psychosocial development of the newborn infant. Using Erikson's psychosocial development theory, the nurse instructs the mother to take which measure? 1. Allow the newborn infant to signal a need. 2. Anticipate all needs of the newborn infant. 3. Attend to the newborn infant immediately when crying. 4. Avoid the newborn infant during the first 10 minutes of crying.

1. Allow the newborn infant to signal a need.

A 4-year-old child diagnosed with leukemia is hospitalized for chemotherapy. The child is fearful of the hospitalization. Which nursing intervention should be implemented to alleviate the child's fears? 1. Encourage the child's parents to stay with the child. 2. Encourage play with other children of the same age. 3. Advise the family to visit only during the scheduled visiting hours. 4. Provide a private room, allowing the child to bring favorite toys from home.

1. Encourage the child's parents to stay with the child.

The nurse is caring for a 14-year-old girl who is hospitalized and has been placed in traction using Crutchfield tongs. The child is having difficulty adjusting to the prolonged hospital confinement. Which nursing action would be appropriate to meet the child's needs? 1. Let the child wear her own clothing when friends visit. 2. Allow the child to have her hair dyed if the parent agrees. 3. Allow the child to play loud music in the hospital room. 4. Allow the child to keep the shades closed and the room darkened at all times

1. Let the child wear her own clothing when friends visit.

Which interventions are appropriate for the care of an infant? Select all that apply. 1. Provide swaddling. 2. Talk in a loud voice. 3. Provide the infant with a bottle of juice at nap time. 4. Hang mobiles with black and white contrast designs. 5. Caress the infant while bathing or during diaper changes. 6. Allow the infant to cry for at least 10 minutes before responding.

1. Provide swaddling. 4. Hang mobiles with black and white contrast designs. 5. Caress the infant while bathing or during diaper changes.

The nurse notes that a 6-year-old child does not recognize that objects exist even when the objects are outside of the visual field. Based on this observation, which action should the nurse take? 1. Report the observation to the health care provider. 2. Move the objects in the child's direct field of vision. 3. Teach the child how to visually scan the environment. 4. Provide additional lighting for the child during play activities.

1. Report the observation to the health care provider.

A parent of a 3-year-old tells a clinic nurse that the child is rebelling constantly and having temper tantrums. Using Erikson's psychosocial development theory, which instructions should the nurse provide to the parent? Select all that apply. 1. Set limits on the child's behavior. 2. Ignore the child when this behavior occurs. 3. Allow the behavior, because this is normal at this age period. 4. Provide a simple explanation of why the behavior is unacceptable. 5. Punish the child every time the child says "no" to change the behavior.

1. Set limits on the child's behavior. 4. Provide a simple explanation of why the behavior is unacceptable.

The nurse is describing Piaget's cognitive developmental theory to pediatric nursing staff. The nurse should tell that staff that which child behavior is characteristic of the formal operations stage? 1. The child has the ability to think abstractly. 2. The child begins to understand the environment. 3. The child is able to classify, order, and sort facts. 4. The child learns to think in terms of past, present, and future.

1. The child has the ability to think abstractly.

A 1-year-old child with hypospadias is scheduled for surgery to correct this condition. While preparing the nursing care plan for this child, which factor should the nurse take into consideration? 1. This surgery is taking place at a time when fears of separation are great. 2. This surgery is taking place at a time when sibling rivalry will cause regression to occur. 3. This surgery is taking place at a time when concern over size and function of the penis is present. 4. This surgery is taking place at a time when embarrassment about voiding irregularities is common.

1. This surgery is taking place at a time when fears of separation are great.

The nurse is caring for a 4-year-old child. When experiencing pain, the nurse anticipates which about the child? Select all that apply. 1. Views pain as a punishment 2. Verbalizes the reason for the pain 3. Blames someone else for the pain 4. Believes pain will disappear magically 5. Fears losing control during the painful episode 6. Will be able to explain the sequence of events leading to the pain

1. Views pain as a punishment 3. Blames someone else for the pain 4. Believes pain will disappear magically

The nurse is caring for a child dying from cancer. Physical signs that the child is approaching death include: 1 rapid pulse. 2 change in respiratory pattern. 3 sensation of cold, although body feels hot. 4 loss of hearing followed by loss of other senses.

2 In the final hours of life the respiratory pattern may become labored, with periods of apnea. The pulse becomes weak and slowed. There is a sensation of heat, although the body feels cold. Hearing is the last sense to fail.

Several nurses tell their nursing supervisor that they want to be able to attend the funeral of a child for whom they had cared. They say they felt especially close to both the child and the family. The supervisor should recognize that attending the funeral is: 1 appropriate because families expect this expression of concern. 2 appropriate because it can assist in the resolution of personal grief. 3 inappropriate because it is unprofessional. 4 inappropriate because it increases burnout.

2 Nurses should attend the funeral of a child if they felt closeness with the family. This will help the nurses grieve and gain closure. Families may or may not expect this expression of concern. The behavior is appropriate if a relationship existed between the nurses and family. This may prevent burnout.

A nursing professor is teaching a group of graduate nurses about anticipatory grief reactions. What information should be included in this presentation? 1 It may last for weeks or up to a few months. 2 It can happen when the death is expected. 3 It happens more than 1 year after the death. 4 It can result in feelings of excessive loneliness.

2 Nurses should be able to identify family members having anticipatory grief reactions. When death is expected, family members may experience anticipatory grief. Anticipatory grief may be manifested in varying behaviors and intensities and may include denial, anger, depression, and other psychological and physical symptoms. Anticipatory grief lasts while the loved one is preparing to die. There is no standard time frame for anticipatory grief. Complicated grief reactions are seen more than 1 year after the loss, whereas anticipatory grief happens before the death. The patient with complicated grief can display symptoms such as unusual sleep disturbances and feelings of excessive loneliness or emptiness.

The nurse is working with a family who has a child with a chronic illness. The nurse notices that the parents watch television and play on their electronic devices instead of spending time with the child. The parents are frequently reprimanding the child. What should the nurse interpret from these findings? The parents: 1 Are being overprotective. 2 Are displaying rejection. 3 Are in denial of the illness. 4 Have gradual acceptance.

2 The parents' reaction of detaching themselves from the child and constantly rebuking the child indicates rejection. However, even though the parents are rejecting the child, they may still meet the child's physical needs. When parents do not let their children learn new skills for fear of injury, this indicates overprotection. Pretending that the child does not have a problem indicates denial. Placing necessary restrictions on the child indicates gradual acceptance.

The parents of a terminally ill child are overwhelmed with the volume of telephone calls asking about the child's condition. What suggestion should the nurse give to the family to make communication easier? 1 "Allow open visiting hours for all family members and the child's friends." 2 "Designate one family member or friend to relay information to everyone else." 3 "Instruct the family to create a website for information and to post updates." 4 "Tell the parents that answering calls and e-mails will help keep them busy."

2 To minimize stress on parents, the family should designate one family member or friend to relay information to others. Open visiting hours would increase the workload of the staff and interfere with care. In addition, the parents would not get any rest because of the continual need to entertain friends and family. Suggesting that the family create a website to post updates or asking parents to answer calls to keep busy may not be helpful. The parents do not need more work to do while dealing with the child's condition.

The nurse is caring for a child who is in the terminal stage of acute lymphocytic leukemia. What should be the goals of palliative care in the child? Select all that apply. 1 To assess the severity of disease symptoms 2 To provide effective pain management 3 To strive for the best possible quality of life 4 To mandate that the family join in the care 5 To support the family and the child

2, 3, 5 Palliative care focuses on relieving pain and managing symptoms related to a terminal disease. It also addresses the psychological, social, and spiritual problems of children living with terminal conditions. The goal of palliative care is to achieve the best quality of life for patients and their families, consistent with their values. Palliative care seeks to improve the quality of life for a patient with a terminal condition. The nurse will assess the severity of the child's symptoms, but this is not the goal of palliative care. The nurse should encourage family members to participate in the child's care but should not mandate their help.

A child is brought to the clinic for a routine check-up. The parent informs the nurse that the child wants to go to school, but the parent does not want the child to go. What other characteristics in the parent would indicate that the parent is overprotective? Select all that apply. 1 Sets same rules for the child and the siblings 2 Continually helps child even if the child is capable 3 Helps the child to learn new skills to be independent 4 Sets high goals without understanding child's abilities 5 Puts restrictions on child's play due to fear of injury

2, 4, 5 A nurse should be able to identify parents who tend to be overprotective about their child with disabilities. The parent may continually help the child even when the child is capable of doing the task. Setting very high or low goals without understanding the child's capabilities makes the child feel incapable and inadequate. Putting restrictions on their child's play due to fear of injury indicates that the parent is too concerned about the child. Setting same rules for all children is not suggestive of an overprotective attitude in the parent. A parent who is not overprotective of the child helps the child to learn skills to be independent.

The mother of a 16-year-old tells the nurse that she is concerned because her child sleeps about 8 hours every night and until noon every weekend. Which nursing response is most appropriate? 1. "The child should not be staying up so late at night." 2. "Adolescents need that amount of sleep every night." 3. "If the child eats properly, that should not be happening." 4. "The child probably is anemic and should eat more foods containing iron."

2. "Adolescents need that amount of sleep every night."

The nurse is caring for a 4-year-old child with human immunodeficiency virus (HIV) infection. The nurse should expect which statement that is aligned with the psychosocial expectations of this age? 1. "Being sick is scary." 2. "I know it hurts to die." 3. "I know I will be healthy soon." 4. "I know I am different than other kids."

2. "I know it hurts to die."

The clinic nurse has provided instructions about dental care for toddlers to the mother of a 2-year-old child. Which statement, if made by the mother, indicates a need for further instruction? 1. "It is best to substitute sweets or snacks with food items such as cheese." 2. "Proper dental care is not necessary for a toddler until the permanent teeth erupt." 3. "My child should have the first dental exam at some point after the second birthday." 4. "I do not need to be concerned if the child swallows some toothpaste while brushing the teeth."

2. "Proper dental care is not necessary for a toddler until the permanent teeth erupt."

A 2-year-old child has been admitted to the hospital for management of pneumonia. The child is placed in an oxygen tent. Taking into consideration the child's age and developmental level and the treatment being administered, which statement is appropriate for the nurse to make to the parents? 1. "He can play in the tent with his blocks and plush stuffed animals." 2. "You can sit next to him and hold his hand through the tent, but he needs to remain inside of it." 3. "At his age, separation anxiety is high, so bringing in the wool blanket that he usually sleeps with is a good idea." 4. "Before you leave for the night, it is a good idea to rock him to sleep. He can be out of the tent for up to 60 minutes without any consequences."

2. "You can sit next to him and hold his hand through the tent, but he needs to remain inside of it."

Which car safety device should be used for a child who is 8 years old and 4 feet tall? 1. Seat belt 2. Booster seat 3. Rear-facing convertible seat 4. Front-facing convertible seat

2. Booster seat

The nurse at a well-baby clinic is providing nutrition instructions to the mother of a 1-month-old infant. What instruction should the nurse give to the mother? 1. Introduce strained fruits one at a time. 2. Breast milk or formula is the main food. 3. Introduce strained vegetables one at a time. 4. Offer rice cereal mixed with breast milk or formula.

2. Breast milk or formula is the main food.

The nurse is monitoring a 3-month-old infant for signs of increased intracranial pressure. On palpation of the fontanels, the nurse notes that the anterior fontanel is soft and flat. On the basis of this finding, which nursing action is most appropriate? 1. Increase oral fluids. 2. Document the finding. 3. Notify the health care provider (HCP). 4. Elevate the head of the bed to 90 degrees.

2. Document the finding.

The nurse assesses the vital signs of a 12-month-old infant with a respiratory infection and notes that the respiratory rate is 35 breaths/minute. On the basis of this finding, which action is most appropriate? 1. Administer oxygen. 2. Document the findings. 3. Notify the health care provider. 4. Reassess the respiratory rate in 15 minutes.

2. Document the findings.

The mother of a toddler informs the nurse that her child has frequent temper tantrums. The nurse should instruct the mother to implement which measure to deal with the temper tantrums 1. Restrain the child. 2. Ignore the behavior. 3. Leave the child unattended. 4. Allow the child to bang his head.

2. Ignore the behavior.

The nurse is observing a caregiver minimize misbehavior when a child is playing with an excessively noisy toy. The nurse recognizes that further instruction is needed about the appropriate way to do this if the caregiver takes which action? 1. Tells the child, "Put that toy down." 2. Instructs the child, "Don't touch that toy." 3. Interacts with the child in a quiet, calm voice. 4. Offers the child a quiet toy in exchange for the noisy one.

2. Instructs the child, "Don't touch that toy."

The nurse is assigned to care for a hospitalized toddler. The nurse plans care, knowing that what should be the highest priority? 1. Providing a consistent caregiver 2. Protecting the toddler from injury 3. Adapting the toddler to the hospital routine 4. Allowing the toddler to participate in play and diversional activities

2. Protecting the toddler from injury

The nurse is preparing to perform a pediatric physical examination. The child refuses to sit on the examining table, screams when the nurse attempts to perform the assessment, and does not make eye contact. What is the most appropriate initial nursing action? 1. Refrain from complimenting the child. 2. Talk to the parent while ignoring the child. 3. Offer a prolonged explanation about the assessment. 4. Use a demanding approach when discussing expected behavior.

2. Talk to the parent while ignoring the child.

A 10-year-old child has been diagnosed with type 1 diabetes mellitus, and the nurse prepares to educate the family. The child is very active socially and often is away from the parents. Which is the best focus of the nurse's teaching for this client? 1. The parents are instructed to always be available to monitor the child's insulin requirements. 2. The child is taught how to monitor insulin requirements and how to self-administer the insulin. 3. All of the friends and family involved with the child's activities should be involved in monitoring the child's insulin requirements. 4. The child's schoolteacher needs instruction on how to assist the child to monitor insulin requirements and how to oversee the child's self-administration of insulin.

2. The child is taught how to monitor insulin requirements and how to self-administer the insulin.

The nurse is evaluating the developmental level of a 2-year-old. Which does the nurse expect to observe in this child? 1. Uses a fork to eat 2. Uses a cup to drink 3. Pours own milk into a cup 4. Uses a knife for cutting food

2. Uses a cup to drink

The nurse educator is preparing to conduct a teaching session for the nursing staff regarding the theories of growth and development and plans to discuss Kohlberg's theory of moral development. What information should the nurse include in the session? Select all that apply. 1. Individuals move through all 6 stages in a sequential fashion. 2. Moral development progresses in relationship to cognitive development. 3. A person's ability to make moral judgments develops over a period of time. 4. The theory provides a framework for understanding how individuals determine a moral code to guide their behavior. 5. In stage 1 (punishment-obedience orientation), children are expected to reason as mature members of society. 6. In stage 2 (instrumental-relativist orientation), the child conforms to rules to obtain rewards or have favors returned.

2. Moral development progresses in relationship to cognitive development. 3. A person's ability to make moral judgments develops over a period of time. 4. The theory provides a framework for understanding how individuals determine a moral code to guide their behavior. 6. In stage 2 (instrumental-relativist orientation), the child conforms to rules to obtain rewards or have favors returned.

A 14-year-old patient with Ewing's sarcoma is scheduled for an amputation of the right leg. The patient has been grieving about the loss ever since the health care provider prescribed the surgery. What type of grief does the nurse identify in this patient? 1 Acute grief 2 Universal loss 3 Anticipatory grief 4 Complicated grief

3 Because the amputation has not occurred, the patient's reaction is caused by anticipation of a future loss. This is a healthy response to the anticipated loss of the leg. It will help foster healthy grieving. Acute grief is a definite syndrome characterized by psychological and somatic symptoms. It is characterized by emotional distancing in relationships with others, accompanied by erratic responses of irritability, hostility, and anger. Universal loss does not describe the emotions of grieving before an actual loss. Some people feel constantly upset and preoccupied with a person who has died, to the point where their relationships and work suffer for months on end. Such a reaction is known as complicated grief.

The nurse is caring for an 8-year-old child who has a chronic illness. The child has a tracheostomy, and a parent is rooming-in during this hospitalization. The parent insists on providing almost all of the child's care and tells the nurses how to care for the child. When planning the child's care, the primary nurse should recognize that the parent is: 1 controlling and demanding. 2 assuming the nurse's role. 3 the expert in care of the child. 4 not allowing nurses to function independently.

3 Because these parents care for this child with complex health needs at home, they are most familiar with the care requirements and routine. The nurse's role includes assessment and evaluation, not just the implementation phase. The nurse recognizes that the philosophy of family-centered care states that the parents are the experts in the care of their child. The nurse functions collaboratively with the family.

The parents of a 10-year-old child diagnosed with terminal cancer ask the nurse not to inform their child about the condition. What would be the most appropriate response of the nurse? 1 Report the situation to the appropriate child welfare organization. 2 Ignore the parents' request and tell the child about the condition. 3 Inform the parents that being honest with the child is important. 4 Tell the parents that treatment cannot be initiated until the child is informed.

3 Often, parents ask health care providers not to reveal the prognosis to the terminally ill child. This is a difficult situation for health care providers. Although children do not understand that they are going to die, they are in a position to understand that something is seriously wrong with them. Therefore the nurse should make the parents understand that being honest with the child is important. Being honest creates an open environment where feelings may be shared by the entire family. This often encourages the parents to communicate openly with the child. The nurse is not required to report to the child welfare organization at this stage. The parents' decision should not be ignored; rather, the parents should be encouraged to talk openly with the child. The nurse cannot withhold treatment.

A preschooler is diagnosed with terminal stage bronchial carcinoma. What would the child consider the most probable reason for the condition? 1 The child will blame the parents for acquiring the disease. 2 The child will blame the grandparents for acquiring this condition. 3 The child would consider it to be a punishment for his or her own actions. 4 The child would understand the pathophysiology of the disease.

3 Preschoolers tend to believe that their thoughts or actions are sufficient to cause death or disease. Therefore, the child is most likely to think that the disease is a punishment for the child's action. This may result in the child feeling guilty. The nurse should help parents to understand this kind of reaction of their child and encourage them to be with the child. The child will not blame the parents or grandparents for the condition. The child is too young to understand the pathophysiology of the disease.

The parents of a terminally ill child are speaking with the nurse to learn about palliative care and how it differs from hospice care. What should the nurse tell them? "Palliative care: 1 Requires that a patient be a Medicare recipient." 2 Does not provide care for hospitalized patients." 3 Is provided when no effective cure is available." 4 Does not provide spiritual support to the family."

3 The terms palliative care and hospice care are associated with end-of-life care. Palliative care may be provided when no cure is available. Palliative care seeks to prevent, relieve, reduce, or soothe the symptoms of disease or disorder without effecting cure. Hospice care supports the patient and the family during the dying process (medical prognosis of 6 months or less) and the surviving family members through the process of bereavement. It provides total care for the patient in the hospital, home, and nursing home when the patient's disease is unresponsive to the curative treatment. To qualify for hospice care, the patient does not need to be a Medicare recipient. Most insurance programs cover hospice care. However, for palliative care, the patient needs to be a Medicare recipient. Palliative care can be provided to patients in a variety of settings, including hospitals. Both palliative care and hospice care include spirituality in order to provide holistic care. Nurses need to familiarize themselves with the attitudes and needs of various religious groups so that they can address the spiritual needs of their patients, families, and other members.

Which behaviors would characterize a child adequately coping with a chronic illness? Select all that apply. 1 Behavior problems at home and school 2 Negative attitudes about his or her condition 3 Acceptance of his or her limitations and coping accordingly 4 Taking responsibility for his or her own physical care 5 Identification with other similarly affected people

3, 4, 5 There are two ways of coping with the illness. Children with healthy coping mechanisms accept their limitations and assume responsibility for their care. These children may also identify with other similarly affected people. Maladaptive coping is evident in behavior problems at home and school. These children may also have a negative attitude toward the illness and begin acting out.

The mother of an 8-year-old child tells the clinic nurse that she is concerned about the child because the child seems to be more attentive to friends than anything else. Using Erikson's psychosocial development theory, the nurse should make which response? 1. "You need to be concerned." 2. "You need to monitor the child's behavior closely." 3. "At this age, the child is developing his own personality." 4. "You need to provide more praise to the child to stop this behavior."

3. "At this age, the child is developing his own personality."

A 6-month-old infant is admitted to the hospital. The nurse weighs the infant and notes that the infant's weight is 14 pounds. Which statement by the mother indicates that further teaching is needed? 1. "His weight for his age is just right." 2. "I am so glad he is gaining the correct amount of weight for his age." 3. "I will have to increase his milk intake because he is not gaining enough weight." 4. "He weighed 7 pounds when he was born so he is at the correct weight for his age.

3. "I will have to increase his milk intake because he is not gaining enough weight."

The nurse is instructing the caregiver of a child about reprimanding the child. The nurse recognizes that additional teaching is needed if the caregiver makes which statement to the child? 1. "I like it when you obey." 2. "I need you to listen to me." 3. "You need to stop hitting your sister." 4. "I don't like it when you hit your sister."

3. "You need to stop hitting your sister."

The nurse at a well-baby clinic is assessing the language and communication developmental milestones of a 4-month-old infant. On the basis of the age of the infant, what should the nurse expect to note as the highest-level developmental milestone? 1. Cooing sounds 2. Use of gestures 3. Babbling sounds 4. Increased interest in sounds

3. Babbling sounds

The nurse is developing a plan of care for a 4-year-old child scheduled for a renal biopsy. What developmental characteristic of this child should the nurse consider? 1. Masturbation is common in this age group. 2. Body image may be a concern for the child. 3. Fears of mutilation may be present in the child. 4. The urination pattern will cause embarrassment for the child.

3. Fears of mutilation may be present in the child.

A mother tells the nurse in a pediatrician's office that she is concerned because her children must let themselves into the house after school each day while she is at work. The nurse explores which suggestion with the mother to decrease the children's sense of isolation and fear? 1. Instruct the children never to cook. 2. Let the children play in neighborhood homes. 3. Find community after-school programs or activities. 4. Have the children call the mother at work every hour.

3. Find community after-school programs or activities.

An infant is being seen in the pediatrician's office for a 2-month-old well-child visit. The nurse encourages the mother to allow the infant to suck on a pacifier during a routine immunization. The nurse explains to the mother that the child is at which stage of Piaget's cognitive development? 1. Trust development 2. Autonomy development 3. Sensorimotor development 4. Preconceptual development

3. Sensorimotor development

The parents of a 2-year-old arrive at a hospital to visit their child. The child is in the playroom when the parents arrive. When the parents enter the playroom, the child does not readily approach the parents. Which is the correct interpretation of the behavior? 1. The child is withdrawn. 2. The child is self-centered. 3. The child exhibits detachment. 4. The child has adjusted to the hospital setting.

3. The child exhibits detachment.

The nurse prepares to take the blood pressure of a school-age child. To obtain an accurate measurement, how should the nurse position the blood pressure cuff? 1. One half of the distance between the antecubital fossa and the shoulder 2. One third of the distance between the antecubital fossa and the shoulder 3. Two thirds of the distance between the antecubital fossa and the shoulder 4. One quarter of the distance between the antecubital fossa and the shoulder

3. Two thirds of the distance between the antecubital fossa and the shoulder

The clinic nurse assesses the communication patterns of a 5-month-old infant. Which assessment finding should lead the nurse to determine that the infant is demonstrating the highest level of developmental achievement expected? 1. Coos when comforted 2. Links syllables together 3. Uses monosyllabic babbling 4. Uses simple words such as mama

3. Uses monosyllabic babbling

The nurse is reviewing the plan of care with the parent of a child with Crohn's disease. Which statement made by the parent indicates a need for further education? 1 "This illness is why my child has delayed puberty." 2 "This illness is caused by the intake of fiber-rich food." 3 "This illness has impaired my child's growth and development." 4 "This illness is God's way of testing our religious faith."

4 A parent who states that the child's illness is a trial sent by God to test religious faith is expressing a self-accusatory feeling that indicates a lack of understanding of the condition. Crohn's disease is an autoimmune disorder that affects the gastrointestinal tract, causing inflammation that can result in excessive diarrhea, abdominal pain, and other symptoms. Crohn's disease can stunt growth and weaken bones in children who are going through puberty. Parents are advised to refrain from serving fiber-rich food to such children because these foods are difficult to digest and may block the intestine.

The nurse is caring for a child who is in the final stages of death. What can the nurse do to provide comfort for the parents? 1 Explain to the parents that births and deaths are normal. 2 Inform the family that their child will soon be at peace. 3 Ask the parents to stay away from the child at this time. 4 Have the parents focus on the quality of the child's remaining life.

4 Acknowledging grief is often the first step toward facing the reality of the child's illness. It is important to have the parents spend the time that is left with their child. Such acceptance may help parents focus on the quality of the child's remaining life. The nurse should not be so blunt as to say that births and deaths are normal. It is not showing compassion. Even though the child will be at peace when the child dies, this is not the best response because it does not provide comfort. The nurse should not ask the parents to stay away from their child in their final stages of life.

A 27-year-old patient reports having difficulty sleeping, having a poor appetite, and feeling sad since the death of her spouse. What should the nurse identify in this patient as a normal response to the death of the spouse? 1 Grief 2 Mourning 3 Acceptance 4 Bereavement

4 Bereavement is a process of grief experienced by most people. Examples include feelings of sadness, insomnia, poor appetite, deprivation, and desolation. The grieving person may seek professional help for the relief of symptoms if they interfere with daily activities and do not subside within a few months of the loss. Grief is a normal, appropriate emotional response to an external or consciously recognized loss. Mourning is a term used to describe a person's outward expression of grief. The person experiences emotional detachment from the loved object or person. Acceptance is the stage where the patient has achieved inner and outer peace through a personal victory over fear.

The parents of a child are grieving the loss of their child. The mother of the child has disturbed sleep. The father informs the nurse that the mother can hear their child talk. The mother wants to be left alone and does not speak to any of the family members. What should be the most appropriate response of the nurse? 1 Tell the father that his wife may have mental breakdown in near future. 2 Tell the father that his wife is coping very poorly and needs professional help. 3 Inform the father that his wife has hallucinations and needs psychiatric referral. 4 Inform the father that it is a normal part of grief and would subside over time.

4 Grief reactions vary from person to person and it is required for the healing to occur. It is important for the nurse to inform the husband that hearing the dead person talk and distancing from others is very common reaction. It does not indicate any impending mental breakdown, poor coping, or psychiatric problem. Therefore, the husband should be reassured that his wife is trying to cope with the grief and she is normal.

A child who is terminally ill with bone cancer is in severe pain. Nursing interventions should be based on knowledge that: 1 children tend to be overmedicated for pain. 2 giving large doses of opioids causes euthanasia. 3 narcotic addiction is common in terminally ill children. 4 large doses of opioids are justified when there are no other treatment options.

4 Large doses may be needed because the child has become physiologically tolerant to the drug, requiring higher doses to achieve the same degree of pain control. Continuing studies report that children are consistently undermedicated for pain. The dose is titrated to relieve pain. Addiction refers to a psychological dependence on the medication, which does not happen in terminal care.

The parents of a child who is dying of leukemia are grieving their impending loss. What intervention should the nurse perform to provide effective care for the family? 1 Encourage the use of alcohol and drugs as a way to escape grief. 2 Refer the family to a self-help group to manage their grief. 3 Emphasize that acute grieving is a process that lasts for years. 4 Provide the child and family with time to share memories.

4 The nurse should be very sensitive to the feelings of a grieving family. The nurse should also provide the child and family with the opportunity to review special experiences or memories in their lives. This helps the family members share their emotions and bond with each other. Parents and family members should be advised to seek professional counseling or work with each other to handle grief. The nurse should never encourage people to use alcohol or drugs to escape grief. The nurse can refer the family to a self-help group, but this is not always an effective way to help grieving parents. They may need a professional counselor to help guide them through the grieving process. The nurse should emphasize to the family that acute grieving only lasts for a few weeks to months, but the entire grieving process is a painful process that often takes years to resolve.

The nurse is preparing to care for a 5-year-old who has been placed in traction following a fracture of the femur. The nurse plans care, knowing that which is the most appropriate activity for this child? 1. A radio 2. A sports video 3. Large picture books 4. Crayons and a coloring book

4. Crayons and a coloring book

The student nurse is caring for a 10-year-old child in the terminal stages of acute lymphocytic leukemia. Which instructions should the nursing instructor provide to the student nurse when caring for the child? Select all that apply. "You should: 1 Challenge the child to learn ways to manage her own care." 2 Advise the child's parents to pray to God for recovery." 3 "Not involve yourself in the family's emotional distress." 4 "Urge the parents to accept the truth and spend time together." 5 "Encourage the child's parents to ask for help when they need it."

4, 5 The nurse should encourage child's parents to accept the truth and spend the remainder of the time left making memories with the sick child. The nurse should encourage the child's parents to ask for help when they need it and not try to handle everything alone. The child should not be asked to manage her own care. During terminal illness the child needs to depend on her parents and family more than ever. The nurse should not advise the child's parents to pray to God because it is unethical to force religion on a family. The nurse should not get involved in the family's emotional distress. Instead, the nurse should teach them about ways to manage the child's condition.

The nurse in the well-baby clinic has provided instructions regarding dental care to the mother of a 10-month-old child. Which statement by the mother indicates a need for further instruction? 1. "I need to limit the amount of concentrated sweets." 2. "I need to use fluoride supplements if the water is not fluoridated." 3. "I need to start dental hygiene as soon as the primary teeth erupt." 4. "I can coat a pacifier with honey during the day as long as I do not give my child a bottle at nap or bedtime."

4. "I can coat a pacifier with honey during the day as long as I do not give my child a bottle at nap or bedtime."

The nurse educator is orienting a new nurse to the pediatric unit and is including tips for medication administration. Which statement by the new nurse indicates that the teaching has been effective? 1. "It helps to use magical thinking with the infant age group." 2. "It helps to use magical thinking with the school-age group." 3. "It helps to use magical thinking with the toddler age group." 4. "It helps to use magical thinking with the preschool age group."

4. "It helps to use magical thinking with the preschool age group."

16-year-old client is admitted to the hospital for acute appendicitis and an appendectomy is performed. Which nursing intervention is most appropriate to facilitate normal growth and development postoperatively? 1. Encourage the client to rest and read. 2. Encourage the parents to room in with the client. 3. Allow the family to bring in the client's favorite computer games. 4. Allow the client to interact with others in his or her (adolescent) same age group

4. Allow the client to interact with others in his or her (adolescent) same age group

A 15-year-old is injured and sustains a fractured jaw. The fractured jaw has been surgically wired, and the health care provider (HCP) has prescribed a full liquid diet. Which nursing action would best promote compliance and provide an adequate nutrient value with the full liquid diet for this teenager? 1. Offer chocolate milkshakes between meals. 2. Explain the importance of good nutrition to the teenager. 3. Offer commercial nutritional supplements 4 to 6 times per day. 4. Ask the teenager for food preferences and liquefy these foods using a blender.

4. Ask the teenager for food preferences and liquefy these foods using a blender.

A mother arrives at a clinic with her toddler and tells the nurse that she has a difficult time getting the child to go to bed at night. What measure is most appropriate for the nurse to suggest to the mother? 1. Allow the child to set bedtime limits. 2. Allow the child to have temper tantrums. 3. Avoid letting the child nap during the day. 4. Inform the child of bedtime a few minutes before it is time for bed.

4. Inform the child of bedtime a few minutes before it is time for bed.

The pediatric nurse is caring for a hospitalized toddler. What does the nurse determine is the most appropriate play activity for the toddler? 1. Listening to music 2. Playing peek-a-boo 3. Hand sewing a picture 4. Playing with a push-pull toy

4. Playing with a push-pull toy

The nurse is admitting a 10-month-old infant who is being hospitalized for a respiratory infection. The nurse develops a plan of care for the infant and includes which most appropriate intervention? 1. Keeping the infant as quiet as possible 2. Restraining the infant to prevent dislodging of tubes 3. Placing small toys in the crib to provide stimulation for the infant 4. Providing a consistent routine with touching, rocking, and cuddling throughout hospitalization

4. Providing a consistent routine with touching, rocking, and cuddling throughout hospitalization

The clinic nurse is preparing to explain the concepts of Kohlberg's theory of moral development with a parent. The nurse should tell the parent that which factor motivates good and bad actions for the child at the preconventional level? 1. Peer pressure 2. Social pressure 3. Parents' behavior 4. Punishment and reward

4. Punishment and reward

Which would be the highest expected growth and development occurrences at 9 months of age for an infant who has had appropriate growth assessed at each well-child visit? Select all that apply. 1. Will smile spontaneously 2. Rolls over in both directions 3. Able to sit steadily unsupported 4. Should be able to say "mama" and "dada" 5. Will pull up and stand for several seconds holding on to furniture 6. Will be able to pick up small pieces of food when placed in a high chair

4. Should be able to say "mama" and "dada" 5. Will pull up and stand for several seconds holding on to furniture 6. Will be able to pick up small pieces of food when placed in a high chair

The nurse at a well-baby clinic is assessing the motor development of a 24-month-old child. On the basis of the age of the child, the nurse expects to note what as the highest-level developmental milestone? 1. The child snaps large snaps. 2. The child builds a tower of 2 blocks. 3. The child puts on simple clothes independently. 4. The child opens a door by turning the doorknob.

4. The child opens a door by turning the doorknob.

Which would be the highest expected growth and development occurrence at 12 months of age for an infant who has had appropriate growth assessed at each well-child visit? 1. Imitates sounds 2. Smiles spontaneously 3. Sits steadily unsupported 4. Walks holding on to someone's hand

4. Walks holding on to someone's hand

A 2-month-old breastfed infant is successfully rehydrated with oral rehydration solutions (ORS) for acute diarrhea. What instructions to the mother about breastfeeding should be included by the nurse? A. Continue breastfeeding. B. Stop breastfeeding until breast milk is cultured. C. Stop breastfeeding until diarrhea is absent for 24 hours. D. Express breast milk, and dilute with sterile water before feeding.

A. Continue breastfeeding. Breastfeeding should continue. Culturing the breast milk is not necessary. Breastfeeding can continue along with ORS to replace the continuing fluid loss from the diarrhea. Breast milk should not be diluted.

Adolescent sexuality refers to the thoughts, feelings, and behaviors related to the teen's sexual identity. The most recent research (2009) indicates that 46% of all adolescents have been involved in some kind of sexual activity. The only complete protection from pregnancy and sexually transmitted diseases (STDs) is ____________.

ANS: abstinence Adolescents should be encouraged that there is nothing wrong with abstaining from sexual activity. Adolescents who engage in sexual activity at a young age are more likely to participate in other high-risk behaviors such as alcohol and drug use. Adolescents who demonstrate high self-esteem are more likely to delay sexual intercourse.

Adolescents' eyes and ears are fully developed and, with the exception of minor infections, the sensory system remains quite healthy during this period of development. The mother of a 12-year-old complains to the nurse that she is concerned that her daughter frequently needs changes to her corrective lenses. This is a condition known as ___________.

ANS: myopia Myopia (nearsightedness) occurs in early adolescence, between the ages of 11 and 13 and is a normal part of adolescent development

Children have a total of __________ primary (deciduous) teeth that they begin to lose when they are school age.

ANS: 20 twenty

Primary (deciduous) teeth are replaced by permanent teeth. By adulthood the child will have __________ permanent teeth.

ANS: 32 thirty-two

In assessing adolescents using Tanner staging, sexual maturity is rated using _________ distinct stages. (Your answer should appear as a number.)

ANS: 5 Tanner stages of adolescent sexual development describe five distinct stages of sexual maturity rating. There are separate rating scales for males and females, but both use five stages.

A nurse is planning a class for school-age children on obesity. Which percentile does the body mass index (BMI) need to exceed for a child to be assessed as obese?

ANS: 95 95th When intake of food exceeds expenditure, the excess is stored as fat. Obesity is an excessive accumulation of fat in the body and is assessed in children as a BMI that exceeds the 95th percentile for age.

The nurse has just assisted in the delivery of a female infant to first-time parents. The infant is suctioned, dried, and placed skin-to-skin on her mother's chest. This allows for significant interaction between mother and baby and is known as _____________.

ANS: attachment Parent-infant attachment is one of the most important aspects of infant psychosocial development. Initiated immediately after birth, attachment is strengthened by many mutually satisfying interactions between parents and their infant during the first few months of life. Attachment is a sense of belonging or connection with each other.

A new mother calls the pediatrician's office concerned because her newborn has developed a salmon colored, irregularly shaped spot between the eyes. The lesion becomes darker when the baby is crying. This skin lesion is called a(n) ____________.

ANS: salmon patch The nurse can reassure the mother that salmon patches are commonly known as "stork bites" or "angel kisses." These lesions are benign and usually fade during the first year of life. The only treatment necessary is parental education.

Bodily fluids are composed of two elements; water and _____.

ANS: solutes Water is the primary constituent of bodily fluids. An infant's weight is approximately 75% water compared to the adult's weight, which is 55% to 60% water. Solutes are composed of both electrolytes and nonelectrolytes. The body's solutes include sodium, potassium, chloride, calcium, and magnesium.

The primary clinical manifestation of scabies is: a. Edema b. Redness c. Pruritus d. Maceration

ANS: C Feedback A Edema is not observed in scabies. B Redness is not observed in scabies. C Scabies is caused by the scabies mite. The inflammatory response and intense itching occur after the host has become sensitized to the mite. This occurs approximately 30 to 60 days after initial contact. In the previously sensitized person, the response occurs within 48 hours. D Maceration is not observed in scabies.

A type of play that allows children to act out roles and experiences that may have happened to them, that they fear may happen, or that they have observed in others is known as ______ play.

ANS: dramatic This type of play can be spontaneous or guided and often includes medical or nursing equipment. It is especially valuable for children who have had or will have multiple procedures or hospitalizations

____________________ is the leading cause of death in children of every age-group beyond 1 year of age.

ANS: Unintentional injury

Which statement best describes development in infants and children? a. Development, a predictable and orderly process, occurs at varying rates within normal limits. b. Development is primarily related to the growth in the number and size of cells. c. Development occurs in a proximodistal direction with fine muscle development occurring first. d. Development is more easily and accurately measured than growth.

ANS: A Feedback A Development, a continuous and orderly process, provides the basis for increases in the child's function and complexity of behavior. The increases in rate of function and complexity can vary normally within limits for each child. B An increase in the number and size of cells is a definition for growth. C Development proceeds in a proximodistal direction with fine muscle organization occurring as a result of large muscle organization. D Development is a more complex process that is affected by many factors; therefore, it is less easily and accurately measured. Growth is a predictable process with standard measurement methods.

Which assessment findings indicate to the nurse that a child has excess fluid volume? Select all that apply. a. Weight gain b. Decreased blood pressure c. Moist breath sounds d. Poor skin turgor e. Rapid bounding pulse

ANS: A, C, E Feedback Correct: A child with fluid volume excess will have a weight gain, moist breath sounds due to the excess fluid in the pulmonary system, and a rapid bounding pulse. Other signs seen with fluid volume excess are increased blood pressure, edema, and fatigue. Incorrect: Decreased blood pressure and poor skin turgor are signs of fluid volume deficit.

The theorist who viewed developmental progression as a lifelong series of conflicts that need resolution is a. Erikson b. Freud c. Kohlberg d. Piaget

ANS: A Feedback A Erik Erikson viewed development as a series of conflicts affected by social and cultural factors. Each conflict must be resolved for the child to progress emotionally, with unsuccessful resolution leaving the child emotionally disabled. B Sigmund Freud proposed a psychosexual theory of development. He proposed that certain parts of the body assume psychological significance as foci of sexual energy. The foci shift as the individual moves through the different stages (oral, anal, phallic, latency, and genital) of development. C Lawrence Kohlberg described moral development as having three levels (preconventional, conventional, and postconventional). His theory closely parallels Piaget's. D Jean Piaget's cognitive theory interprets how children learn and think and how this thinking progresses and differs from adult thinking. Stages of his theory include sensorimotor, preoperations, concrete operations, and formal operations.

The environment, both physical and psychosocial, is a significant determinate of growth and development outcomes before and after birth. Nurses can assist parents in preventing environmental injury for their 2-year-old toddler by teaching them to avoid the most common sources of exposure. This anticipatory guidance includes teaching related to a. Avoiding sun exposure, secondhand smoke, and lead b. Socioeconomic status, primarily poverty c. Maternal smoking and alcohol intake during pregnancy d. The passing of environmental toxins through breast milk

ANS: A Feedback A Lead can be present in the home and in toys made overseas. Environmental injury can also be the result of mercury, pesticides (flea and tick collars), radon, and exposure to the sun and secondhand smoke. It is important for the nurse to provide health teaching related to these factors. B The nurse is unable to influence socioeconomic status. C It is too late for the nurse to instruct the mother regarding smoking or alcohol intake during pregnancy. This should have been included in prenatal teaching. D It is unlikely that a 2-year-old child will still be breastfeeding.

The management of a child who has just been stung by a bee or wasp should include the application of: a. Cool compresses b. Warm compresses c. Antibiotic cream d. Corticosteroid cream

ANS: A Feedback A Bee or wasp stings are initially treated by carefully removing the stinger, cleansing with soap and water, application of cool compresses, and the use of common household agents such as lemon juice or a paste made with aspirin and baking soda. B Warm compresses are avoided. C Antibiotic cream is unnecessary unless a secondary infection occurs. D Corticosteroid cream is not part of the initial therapy. If a severe reaction occurs, systemic corticosteroids may be indicated.

What nursing assessment and care holds the highest priority in the initial care of a child with a major burn injury? a. Establishing and maintaining the child's airway b. Establishing and maintaining intravenous access c. Inserting a catheter to monitor hourly urine output d. Inserting a nasogastric tube into the stomach to supply adequate nutrition

ANS: A Feedback A Establishing and maintaining the child's airway is always the priority focus for assessment and care. B Establishing intravenous access is the second priority in this situation, after the airway has been established. C Inserting a catheter and monitoring hourly urine output is the third most important nursing intervention. D Nasogastric feedings are not begun initially on a child with major or severe burns. The initial assessment and care focus for a child with major burn injuries is the ABCs.

Impetigo ordinarily results in: a. No scarring b. Pigmented spots c. Slightly depressed scars d. Atrophic white scars

ANS: A Feedback A Impetigo tends to heal without scarring unless a secondary infection occurs. B Hyperpigmentation may occur; however, only in dark skinned children. C No scarring usually occurs. D No scarring usually occurs

When assessing the child with atopic dermatitis, the nurse should ask the parents about a history of: a. Asthma b. Nephrosis c. Lower respiratory tract infections d. Neurotoxicity

ANS: A Feedback A Most children with atopic dermatitis have a family history of asthma, hay fever, or atopic dermatitis, and up to 80% of children with atopic dermatitis have asthma or allergic rhinitis. B Complications of atopic dermatitis relate to the skin. The renal system is not affected by atopic dermatitis. C There is no link between lower respiratory tract infections and atopic dermatitis. D Atopic dermatitis does not have a relationship to neurotoxicity.

A child experiences frostbite of the fingers after prolonged exposure to the cold. Which intervention should the nurse implement first?: a. Rapid rewarming of the fingers by placing in warm water b. Placing the hand in cool water c. Slow rewarming by wrapping in warm cloth d. Using an ice pack to keep cold until medical intervention is possible

ANS: A Feedback A Rapid rewarming is accomplished by immersing the part in well-agitated water at 37.8° C to 42.2° C (100° F to 108° F). B The frostbitten area should be rewarmed as soon as possible to avoid further tissue damage. C Rapid rewarming results in less tissue necrosis than slow thawing. D The frostbitten area should be rewarmed, as soon as possible, to avoid further tissue damage.

When changing an infant's diaper, the nurse notices small bright red papules with satellite lesions on the perineum, anterior thigh, and lower abdomen. This rash is characteristic of: a. Primary candidiasis b. Irritant contact dermatitis c. Intertrigo d. Seborrheic dermatitis

ANS: A Feedback A Small red papules with peripheral scaling in a sharply demarcated area involving the anterior thighs, lower abdomen, and perineum are characteristic of primary candidiasis. B A shiny, parchment-like erythematous rash on the buttocks, medial thighs, mons pubis, and scrotum, but not in the folds, is suggestive of irritant contact dermatitis. C Intertrigo is identified by a red macerated area of sharp demarcation in the groin folds. It can also develop in the gluteal and neck folds. D Seborrheic dermatitis is recognized by salmon-colored, greasy lesions with a yellowish scale found primarily in skin-fold areas or on the scalp.

The depth of a burn injury may be classified as: a. Localized or systemic b. Superficial, superficial partial thickness, deep partial thickness, or full thickness c. Electrical, chemical, or thermal d. Minor, moderate, or major

ANS: B Feedback A These terms refer to the effect of the burn injury. For example, is there a reaction in the area of the burn (localized) or throughout the body (systemic)? B The vocabulary to classify the depth of a burn is superficial, partial thickness, or full thickness. C These terms refer to the cause of the burn injury. D These terms refer to the severity of the burn injury.

What should be included in teaching a parent about the management of small red macules and vesicles that become pustules around the child's mouth and cheek? a. Keep the child home from school for 24 hours after initiation of antibiotic treatment. b. Clean the rash vigorously with Betadine three times a day. c. Notify the physician for any itching. d. Keep the child home from school until the lesions are healed.

ANS: A Feedback A To prevent the spread of impetigo to others, the child should be kept home from school for 24 hours after treatment is initiated. Good handwashing is imperative in preventing the spread of impetigo. B The lesions should be washed gently with a warm soapy washcloth three times a day. The washcloth should not be shared with other members of the family. C Itching is common and does not necessitate medical treatment. Rather, parents should be taught to clip the child's nails to prevent maceration of the lesions. D The child may return to school 24 hours after initiation of antibiotic treatment.

Which activity is most appropriate for developing fine motor skills in the school-age child? a. Drawing b. Singing c. Soccer d. Swimming

ANS: A Feedback A Activities such as drawing, building models, and playing a musical instrument increase the school-age child's fine motor skills. B Singing is an appropriate activity for the school-age child, but it does not increase fine motor skills. C The school-age child needs to participate in group activities to increase both gross motor skills and social skills, but group activities do not increase fine motor skills. D Swimming is an activity that also increases gross motor skills.

Which diet would the nurse recommend to the mother of a child who is having mild diarrhea? a. Rice, potatoes, yogurt, cereal, and cooked carrots b. Bananas, rice, applesauce, and toast c. Apple juice, hamburger, and salad d. Whatever the child would like to eat

ANS: A Feedback A Bland but nutritious foods including complex carbohydrates (rice, wheat, potatoes, cereals), yogurt, cooked vegetables, and lean meats are recommended to prevent dehydration and hasten recovery. B These foods used to be recommended for diarrhea (BRAT diet). These foods are easily tolerated, but the BRAT diet is low in energy, density, fat, and protein. C Fatty foods, spicy foods, and foods high in simple sugars should be avoided. D The child should be offered foods he or she likes but should not be encouraged to eat fatty foods, spicy foods, and foods high in simple sugars.

The nurse advises the mother of a 3-month-old exclusively breastfed infant to a. Start giving the infant a vitamin D supplement. b. Start using an infant feeder and add rice cereal to the formula. c. Start feeding the infant rice cereal with a spoon at the evening feeding. d. Continue breastfeeding without any supplements.

ANS: A Feedback A Breast milk does not provide an adequate amount of dietary vitamin D. Infants who are exclusively breastfed need vitamin D supplements to prevent rickets. B An infant feeder is an inappropriate method of providing the infant with caloric intake. Solid foods are not recommended for a 3-month-old infant. C Rice cereal and other solid foods are contraindicated in a 3-month-old infant. Solid feedings do not typically begin before 4 to 6 months of age. D Because breast milk is not an adequate source of fluoride, infants need to be given a fluoride supplement in addition to a vitamin D supplement.

The ability to mentally understand that 1 + 3 = 4 and 4 - 1 = 3 occurs in which stage of cognitive development? a. Concrete operations b. Formal operations c. Intuitive thought d. Preoperations

ANS: A Feedback A By 7 to 8 years of age, the child is able to retrace a process (reversibility) and has the skills necessary for solving mathematical problems. This stage is called concrete operations. B The formal operations stage deals with abstract reasoning and does not occur until adolescence. C Thinking in the intuitive stage is based on immediate perceptions. A child in this stage often solves problems by random guessing. D In preoperational thinking, the child is usually able to add 1 + 3 = 4 but is unable to retrace the process.

In terms of fine motor development, what should the 7-month-old infant be able to do? a. Transfer objects from one hand to the other. b. Use thumb and index finger in crude pincer grasp. c. Hold crayon and make a mark on paper. d. Release cubes into a cup.

ANS: A Feedback A By age 7 months, infants can transfer objects from one hand to the other, crossing the midline. B The crude pincer grasp is apparent at approximately age 9 months. C The child can scribble spontaneously at age 15 months. D At age 12 months, the child can release cubes into a cup.

An 8-year-old girl tells the nurse that she has cancer because God is punishing her for "being bad." She shares her concern that if she dies, she will go to hell. The nurse should interpret this as being a. A belief common at this age b. A belief that forms the basis for most religions c. Suggestive of excessive family pressure d. Suggestive of a failure to develop a conscience

ANS: A Feedback A Children at this age may view illness or injury as a punishment for a real or imagined mystique. B The belief in divine punishment is common at this age. C The belief in divine punishment is common at this age. D The belief in divine punishment is common at this age.

According to Piaget, the adolescent is in the fourth stage of cognitive development, or period of what? a. Formal operations b. Concrete operations c. Conventional thought d. Postconventional thought

ANS: A Feedback A Cognitive thinking culminates with capacity for abstract thinking. This stage, the period of formal operations, is Piaget's fourth and last stage. B Concrete operations usually develops between ages 7 and 11 years. C Conventional and postconventional thought refer to Kohlberg's stages of moral development. D Conventional and postconventional thought refer to Kohlberg's stages of moral development.

A school nurse is teaching a health class for 5th grade children. The nurse plans to include which statement to best describe growth in the early school-age period? a. Boys grow faster than girls. b. Puberty occurs earlier in boys than in girls. c. Puberty occurs at the same age for all races and ethnicities. d. It is a period of rapid physical growth.

ANS: A Feedback A During the school-age developmental period, boys are approximately 1 inch taller and 2 pounds heavier than girls. B Puberty occurs 1 1/2 to 2 years later in boys, which is developmentally later than puberty in girls (not unusual in 9- or 10-year-old girls). C Puberty occurs approximately 1 year earlier in African-American girls than in white girls. D Physical growth is slow and steady during the school-age years.

The most fatal type of burn in the toddler age-group is a. Flame burn from playing with matches b. Scald burn from high-temperature tap water c. Hot object burn from cigarettes or irons d. Electric burn from electrical outlets

ANS: A Feedback A Flame burns from matches and lighters represent one of the most fatal types of burns in the toddler age-group. B These are all significant causes of burn injury. The child should be protected from these causes by reducing the temperature on the hot water heater in the home, keeping objects such as cigarettes and irons away from children, and placing protective guards over electric outlets when not in use. C These are all significant causes of burn injury. The child should be protected from these causes by reducing the temperature on the hot water heater in the home, keeping objects such as cigarettes and irons away from children, and placing protective guards over electric outlets when not in use. D These are all significant causes of burn injury. The child should be protected from these causes by reducing the temperature on the hot water heater in the home, keeping objects such as cigarettes and irons away from children, and placing protective guards over electric outlets when not in use.

A 14-year-old male seems to be always eating, although his weight is appropriate for his height. The best explanation for this is that a. This is normal because of increase in body mass. b. This is abnormal and suggestive of future obesity. c. His caloric intake would have to be excessive. d. He is substituting food for unfilled needs

ANS: A Feedback A In adolescence, nutritional needs are closely related to the increase in body mass. The peak requirements occur in the years of maximal growth. The caloric and protein requirements are higher than at almost any other time of life. B This describes the expected eating pattern for young adolescents; as long as weight and height are appropriate, obesity is not a concern. C This describes the expected eating pattern for young adolescents; as long as weight and height are appropriate, obesity is not a concern. D This describes the expected eating pattern for young adolescents; as long as weight and height are appropriate, obesity is not a concern.

What is the best response for the nurse to give a parent about contacting the physician regarding an infant with diarrhea? a. "Call your pediatrician if the infant has not had a wet diaper for 6 hours." b. "The pediatrician should be contacted if the infant has two loose stools in an 8-hour period." c. "Call the doctor immediately if the infant has a temperature greater than 100° F." d. "Notify the pediatrician if the infant naps more than 2 hours."

ANS: A Feedback A No urine output in 6 hours needs to be reported because it indicates dehydration. B Two loose stools in 8 hours is not a serious concern. If blood is obvious in the stool or the frequency increases to one bowel movement every hour for more than 8 hours, the physician should be notified. C A fever greater than 101° F should be reported to the infant's physician. D It is normal for the infant who is not ill to nap for 2 hours. The infant who is ill may nap longer than the typical amount.

Which toy is the most developmentally appropriate for an 18- to 24-month-old child? a. A push-and-pull toy b. Nesting blocks c. A bicycle with training wheels d. A computer

ANS: A Feedback A Push-and-pull toys encourage large muscle activity and are appropriate for toddlers. B Nesting blocks are more appropriate for a 12- to 15-month-old child. C A bicycle with training wheels is appropriate for a preschool or young school-age child. D A computer can be appropriate as early as the preschool years.

In terms of gross motor development, what would the nurse expect a 5-month-old infant to do? a. Roll from abdomen to back. b. Roll from back to abdomen. c. Sit erect without support. d. Move from prone to sitting position.

ANS: A Feedback A Rolling from abdomen to back is developmentally appropriate for a 5-month-old infant. B The ability to roll from back to abdomen usually occurs at 6 months old. C Sitting erect without support is a developmental milestone usually achieved by 8 months. D The 10-month-old infant can usually move from a prone to a sitting position.

A 9-year-old girl often comes to the school nurse complaining of stomach pains. Her teacher says she is completing her schoolwork satisfactorily, but lately she has been somewhat aggressive and stubborn in the classroom. The school nurse should recognize this as a. Signs of stress b. Developmental delay c. Physical problem causing emotional stress d. Lack of adjustment to school environment

ANS: A Feedback A Signs of stress include stomach pains or headache, sleep problems, bedwetting, changes in eating habits, aggressive or stubborn behavior, reluctance to participate, or regression to early behaviors. B This child is exhibiting signs of stress. C This child is exhibiting signs of stress. D This child is exhibiting signs of stress.

Which is appropriate play for a 6-month-old infant? a. Pat-a-cake, peek-a-boo b. Ball rolling, hide-and-seek game c. Bright rattles and tactile toys d. Push and pull toys

ANS: A Feedback A Six-month-old children enjoy playing pat-a-cake and peek-a-boo. B Nine-month-old infants enjoy rolling a ball and playing hide-and-seek games. C Four-month-old infants enjoy bright rattles and tactile toys. D Twelve-month-old infants enjoy playing with push and pull toys.

The parents of a child with acid-base imbalance ask the nurse about mechanisms that regulate acid-base balance. Which statement by the nurse accurately explains the mechanisms regulating acid-base balance in children? a. The respiratory, renal, and chemical-buffering systems b. The kidneys balance acid; the lungs balance base c. The cardiovascular and integumentary systems d. The skin, kidney, and endocrine systems

ANS: A Feedback A The acid-base system is regulated by chemical buffering, respiratory control of carbon dioxide, and renal regulation of bicarbonate and secretion of hydrogen ions. B Both the kidneys and the lungs, along with the buffering system, contribute to acid-base balance. Neither system regulates acid or base balances exclusively. C The cardiovascular and integumentary systems are not part of acid-base regulation in the body. D Chemical buffers, the lungs, and the kidneys work together to keep the blood pH within normal range.

A nurse is teaching adolescent boys about pubertal changes. The first sign of pubertal change seen with boys is a. Testicular enlargement b. Facial hair c. Scrotal enlargement d. Voice deepens

ANS: A Feedback A The first sign of pubertal changes in boys is testicular enlargement in response to testosterone secretion, which usually occurs in Tanner stage 2. Slight pubic hair is present and the smooth skin texture of the scrotum is somewhat altered. B During Tanner stages 4 and 5, facial hair appears at the corners of the upper lip and chin. C As testosterone secretion increases, the penis, testes, and scrotum enlarge. D During Tanner stages 4 and 5, rising levels of testosterone cause the voice to deepen.

The parent of 16-month-old Chris asks, "What is the best way to keep Chris from getting into our medicines at home?" The nurse should advise that a. "All medicines should be locked away securely." b. "The medicines should be placed in high cabinets." c. "Chris just needs to be taught not to touch medicines." d. "Medicines should not be kept in the homes of small children."

ANS: A Feedback A The major reason for poisoning in the home is improper storage. Toddlers can climb, unlatch cabinets, and obtain access to high-security places. For medications, only a locked cabinet is safe. B Toddlers can climb by using furniture. High places are not a deterrent to an exploring toddler. C Toddlers are not able to generalize as dangerous all of the different forms of medications that may be available in the home. D This is not feasible. Many parents require medications for chronic illnesses. Parents must be taught safe storage for their home and when they visit other homes

Which action is the primary concern in the treatment plan for a child with persistent vomiting? a. Detecting the cause of vomiting b. Preventing metabolic acidosis c. Positioning the child to prevent further vomiting d. Recording intake and output

ANS: A Feedback A The primary focus of managing vomiting is detection of the cause and then treatment of the cause. B Metabolic alkalosis results from persistent vomiting. Prevention of complications is the secondary focus of treatment. C The child with persistent vomiting should be positioned upright or side-lying to prevent aspiration. D Recording intake and output is a nursing intervention, but it is not the primary focus of treatment.

The nurse is performing a routine assessment on a 14-month-old infant and notes that the anterior fontanel is closed. This should be interpreted as a(n) a. Normal finding b. Questionable finding—infant should be rechecked in 1 month c. Abnormal finding—indicates need for immediate referral to practitioner d. Abnormal finding—indicates need for developmental assessment

ANS: A Feedback A This is a normal finding. The anterior fontanel closes between ages 12 and 18 months. The posterior fontanel closes between 2 and 3 months of age. B Because the anterior fontanel normally closes between ages 12 and 18 months, this is a normal finding, and no further intervention is required. C Because the anterior fontanel normally closes between ages 12 and 18 months, this is a normal finding, and no further intervention is required. D Because the anterior fontanel normally closes between ages 12 and 18 months, this is a normal finding, and no further intervention is required.

A 3-month-old infant born at 38 weeks of gestation will hold a rattle if it is put in her hands, but she will not voluntarily grasp it. The nurse should interpret this as a. Normal development b. Significant developmental lag c. Slightly delayed development as a result of prematurity d. Suggestive of a neurologic disorder such as cerebral palsy

ANS: A Feedback A This is indicative of normal development. Reflexive grasping occurs during the first 2 to 3 months and then gradually becomes voluntary. B The infant is expected to be able to perform this task by age 3 months. If the child's age is corrected because of being 2 weeks preterm, the child is at the midpoint of the range for this developmental task. C The infant is expected to be able to perform this task by age 3 months. If the child's age is corrected because of being 2 weeks preterm, the child is at the midpoint of the range for this developmental task. D The child is age-appropriate. No evidence of neurologic dysfunction is present.

You are working as the triage nurse in a pediatric emergency room. You receive a telephone call from the mother of an adolescent whose front tooth was completely knocked out of his mouth while he was playing soccer. The mother is seeking advice. Which is the appropriate response? Select all that apply. a. Rinse the tooth in lukewarm tap water. b. Place the tooth in saline, milk, or water. c. Scrub the tooth with a disinfectant, such as mouth wash. d. Bring the child to the emergency room within the next hour for the best prognosis.

ANS: A, B Feedback Correct Rinse the tooth in lukewarm tap water—this is a correct response. Place the tooth in saline, milk, or water—this is a correct response. Incorrect The tooth should not be scrubbed, and cleaning agents and disinfectants should be avoided. The prognosis is best if the injury is treated within 30 minutes.

Peer victimization is becoming a significant problem for school-age children and adolescents in the United States. Parents should be educated regarding signs that a child is being bullied. These might include (select all that apply) a. The child spends an inordinate amount of time in the nurse's office. b. Belongings frequently go missing or are damaged. c. The child wants to be driven to school. d. School performance improves. e. The child freely talks about his day.

ANS: A, B, C Feedback Correct Signs that may indicate a child is being bullied are similar to signs of other types of stress and include nonspecific illness or complaints, withdrawal, depression, school refusal, and decreased school performance. Children express fear of going to school or riding the school bus. Very often, children will not talk about what is happening to them. Incorrect These are not indications of bullying.

Which demonstrates the school-age child's developing logic in the stage of concrete operations? Select all that apply. a. The school-age child is able to recognize that 1 lb of feathers is equal to 1 lb of metal. b. The school-age child is able to recognize that he can be a son, brother, or nephew at the same time. c. The school-age child understands the principles of adding, subtracting, and reversibility. d. The school-age child has thinking that is characterized by egocentrism, animism, and centration

ANS: A, B, C Feedback Correct The school-age child understands that the properties of objects do not change when their order, form, or appearance does. Conservation occurs in the concrete operations stage. Comprehension of class inclusion occurs as the school-age child's logic increases. The child begins to understand that a person can be in more than one class at the same time. This is characteristic of concrete thinking and logical reasoning. The school-age child is able to understand principles of adding and subtracting, as well as the process of reversibility, which occurs in the stage of concrete operations. Incorrect This type of thinking occurs in the intuitive thought stage, not the concrete operations stage of development.

Which play patterns does a 3-year-old child typically display? Select all that apply. a. Imaginary play b. Parallel play c. Cooperative play d. Structured play e. Associative play

ANS: A, B, C, E Feedback Correct Children between ages 3 and 5 years enjoy parallel and associative play. Children learn to share and cooperate as they play in small groups. Play is often imitative, dramatic, and creative. Imaginary friends are common around age 3 years. Incorrect Structured play is typical of school-age children.

The nurse is assessing parental knowledge of temper tantrums. Which are true statements about temper tantrums? Select all that apply. a. Temper tantrums are a common response to anger and frustration in toddlers. b. Temper tantrums often include screaming, kicking, throwing things, and head banging. c. Parents can effectively manage temper tantrums by giving in to the child's demands. d. Children having temper tantrums should be safely isolated and ignored. e. Parents can learn to anticipate times when tantrums are more likely to occur.

ANS: A, B, D, E Feedback Correct Temper tantrums are a common response to anger and frustration in toddlers. They occur more often when toddlers are tired, hungry, bored, or excessively stimulated. A nap before fatigue or a snack if mealtime is delayed will be helpful in alleviating the times when tantrums are most likely to occur. Tantrums may include screaming, kicking, throwing things, biting themselves, or banging their head. Effective management of tantrums includes safely isolating and ignoring the child. Incorrect The child should learn that nothing is gained by having a temper tantrum. Giving in to the child's demands only increases the behavior.

A nurse is teaching parents about prevention of diaper dermatitis. Which should the nurse include in the teaching plan?( Select all that apply.): a. Clean the diaper area gently after every diaper change with a mild soap. b. Use a protective ointment to clean dry intact skin. c. Use a steroid cream after each diaper change. d. Use rubber or plastic pants over the diaper. e. Wash cloth diapers in hot water with a mild soap and double rinse.

ANS: A, B, E Feedback Correct: Prompt, gentle cleaning with water and mild soap (e.g., Dove, Neutrogena Baby Soap) after each voiding or defecation rids the skin of ammonia and other irritants and decreases the chance of skin breakdown and infection. A bland, protective ointment (e.g., A&D, Balmex, Desitin, zinc oxide) can be applied to clean, dry, intact skin to help prevent diaper rash. If cloth diapers are laundered at home, the parents should wash them in hot water, using a mild soap and double rinsing. Incorrect: Occlusion increases the risk of systemic absorption of a steroid; thus steroid creams are rarely used for diaper dermatitis because the diaper functions as an occlusive dressing. Rubber or plastic pants increase skin breakdown by holding in moisture and should be used infrequently. A steroid cream is not recommended.

Hearing seems to be relatively acute, even at birth, as shown by reflexive generalized reaction to noise. All newborns should undergo hearing screening at birth, before hospital discharge. In addition, assessment for hearing deficits should take place at every well-baby visit. Risk factors for hearing loss include (select all that apply) a. Structural abnormalities of the ear b. Family history of hearing loss c. Alcohol or drug use by the mother during pregnancy d. Gestational diabetes e. Trauma

ANS: A, B, E Feedback Correct Structural abnormalities of the ear, a family history of hearing loss, and trauma are risk factors for hearing loss. Other risk factors include persistent otitis media and developmental delay. The American Academy of Pediatrics suggests that infants who demonstrate hearing loss be eligible for early intervention and specialized hearing and language services. Incorrect Prenatal alcohol or drug intake and gestational diabetes are not risk factors for hearing loss in the infant.

Where do the lesions of atopic dermatitis most commonly occur in the infant? (Select all that apply.): a. Cheeks b. Buttocks c. Extensor surfaces of arms and legs d. Back e. Trunk

ANS: A, C, E Feedback Correct: The lesions of atopic dermatitis are generalized in the infant. They are most commonly on the cheeks, scalp, trunk, and extensor surfaces of the extremities. Incorrect: These lesions are not typically on the back or the buttocks.

A nurse has completed a teaching session for parents about "baby-proofing" the home. Which statements made by the parents indicate an understanding of the teaching? Select all that apply. a. "We will put plastic fillers in all electrical plugs." b. "We will place poisonous substances in a high cupboard." c. "We will place a gate at the top and bottom of stairways." d. "We will keep our household hot water heater at 130 degrees." e. "We will remove front knobs from the stove."

ANS: A, C, E Feedback Correct By the time babies reach 6 months of age, they begin to become much more active, curious, and mobile. Putting plastic fillers on all electrical plugs can prevent an electrical shock. Putting gates at the top and bottom of stairways will prevent falls. Removing front knobs form the stove can prevent burns. Incorrect Poisonous substances should be stored in a locked cabinet not in a cabinet that children can reach when they begin to climb. The household hot water heater should be turned down to 120 degrees or less.

Which interventions are appropriate for preventing childhood obesity? Select all that apply. a. Establish consistent times for meals and snacks. b. Eliminate all snacks. Eat three nutritious meals a day. c. Teach the family and child how to select foods and prepare foods. d. Encourage schools to provide snack machines with popcorn, cookies, and diet soda. e. Limit computer and television time.

ANS: A, C, E Feedback Correct Preventing obesity includes encouraging families to establish consistent times for meals and snacks and discouraging between-meal eating. Parents and children also need to be taught how to select and prepare healthful foods. Because snacks are an important aspect in childhood nutrition, nutritious snacks should be identified. School-age children usually require a healthful snack after school and in the evening. A child who spends time with social media has less interest in physical activity and going outdoors. Incorrect Snacks are an important aspect in childhood nutrition. Nutritious snacks should be identified, not eliminated. Healthy snack options include fruit, popcorn, nuts, and yogurt, not cookies and diet soda. In schools with snack machines, children may use their lunch money to purchase high-calorie snacks versus a nutritious lunch.

Motor vehicle injuries are a significant threat to young children. Knowing this, the nurse plans a teaching session with a toddler's parents on car safety. Which will she teach? Select all that apply. a. Secure in a rear-facing, upright car safety seat. b. Place the car safety seat in the rear seat, behind the driver's seat. c. Harness safety straps should fit snugly. d. Place the car safety seat in the front passenger seat equipped with an airbag. e. After the age of 2 years, toddlers can be placed in a forward-facing car seat.

ANS: A, C, E Feedback Correct Toddlers should be secured in a rear-facing, upright, approved car safety seat. Harness straps should be adjusted to provide a snug fit. Incorrect The car safety seat should be placed in the middle of the rear seat. Children younger than 13 years should not ride in a front passenger seat that is equipped with an airbag.

Injuries claim many lives during adolescence. Which factors contribute to early adolescents engaging in risk-taking behaviors? Select all that apply. a. Peer pressure b. A desire to master their environment c. Engagement in the process of separation from their parents d. A belief that they are invulnerable e. Impulsivity

ANS: A, D, E Feedback Correct Peer pressure (including impressing peers) is a factor contributing to adolescent injuries. During early to middle adolescence, children feel that they are exempt from the consequences of risk-taking behaviors; they believe that negative consequences only happen to others. Feelings of invulnerability ("It can't happen to me") are evident in adolescence. Impulsivity places adolescents in unsafe situations. Incorrect Mastering the environment is the task of young school-age children. Emancipation is a major issue for the older adolescent. The process is accomplished as the teenager gains an education or vocational training

Which expected outcome is developmentally appropriate for a hospitalized 4-year-old child? a. The child will be dressed and fed by the parents. b. The child will independently ask for play materials or other personal needs. c. The child will be able to verbalize an understanding of the reason for the hospitalization. d. The child will have a parent stay in the room at all times.

ANS: B Feedback A Parents need to foster appropriate developmental behavior in the 4-year-old child. Dressing and feeding the child do not encourage independent behavior. B Erikson identifies initiative as a developmental task for the preschool child. Initiating play activities and asking for play materials or assistance with personal needs demonstrates developmental appropriateness. C A 4-year-old child cannot be expected to cognitively understand the reason for hospitalization. Expecting the child to verbalize an understanding for hospitalization is an inappropriate outcome. D Parents staying with the child throughout a hospitalization is an inappropriate outcome. Although children benefit from parental involvement, parents may not have the support structure to stay in the room with the child at all times.

A preschool aged child will be receiving immunizations. Which statement identifies an appropriate level of language development for a 4-year-old child? a. The child has a vocabulary of 300 words and uses simple sentences. b. The child uses correct grammar in sentences. c. The child is able to pronounce consonants clearly. d. The child uses language to express abstract thought.

ANS: B Feedback A Simple sentences and a 300-word vocabulary are appropriate for a 2-year-old child. B The 4-year-old child is able to use correct grammar in sentence structure. C The 4-year-old child typically has difficulty in pronouncing consonants. D The use of language to express abstract thought is developmentally appropriate for the adolescent.

Which developmental assessment instrument is appropriate to assess a 5-year-old child? a. Brazelton Behavioral Scale b. Denver Developmental Screening Test II (DDST-II) c. Dubowitz Scale d. New Ballard Scale

ANS: B Feedback A The Brazelton Behavioral Scale is used for newborn assessment. B The DDST-II is used for infants and children between birth and 6 years of age. C The Dubowitz Scale is used for estimation of gestational age. D The New Ballard Scale is used for newborn screening.

Which is the preferred site for administration of the Hib vaccine to an infant? a. Deltoid b. Anterolateral thigh c. Upper, outer aspect of the arm d. Dorsal gluteal region

ANS: B Feedback A The deltoid muscle is not used for infants. B The anterolateral thigh is the preferred site for intramuscular administration of vaccines for infants. C Subcutaneous injections can be given in the upper arm. The HIB vaccine is given by the intramuscular route. D The dorsal gluteal site is never used for vaccines.

Which child is most likely to be frightened by hospitalization? a. A 4-month-old infant admitted with a diagnosis of bronchiolitis b. A 2-year-old toddler admitted for cystic fibrosis c. A 9-year-old child hospitalized with a fractured femur d. A 15-year-old adolescent admitted for abdominal pain

ANS: B Feedback A Young infants are not as likely to be as frightened as toddlers by hospitalization because they are not as aware of the environment. B Toddlers are most likely to be frightened by hospitalization because their thought processes are egocentric, magical, and illogical. They feel very threatened by unfamiliar people and strange environments. C The 9-year-old child's cognitive ability is sufficient enough for the child to understand the reason for hospitalization. D The 15-year-old adolescent has the cognitive ability to interpret the reason for hospitalization.

Which nursing assessment is applicable to the care of a child with herpetic gingivostomatitis? a. Comparison of range of motion for the upper and lower extremities b. Urine output, mucous membranes, and skin turgor c. Growth pattern since birth d. Bowel elimination pattern

ANS: B Feedback A An oral herpetic infection does not affect joint function. B The child with herpetic gingivostomatitis is at risk for deficient fluid volume. Painful lesions on the mouth make drinking unpleasant and undesirable, with subsequent dehydration becoming a real danger. C Herpetic gingivostomatitis is not a chronic disorder that would affect the child's long-term growth pattern. D Although constipation could be caused by dehydration, it is more important to assess urine output, skin turgor, and mucous membranes to identify dehydration before constipation is a problem.

Parents of a child with lice infestation should be instructed carefully in the use of antilice products because of which potential side effect? a. Nephrotoxicity b. Neurotoxicity c. Ototoxicity d. Bone marrow depression

ANS: B Feedback A Antilice products are not known to be nephrotoxic. B Because of the danger of absorption through the skin and potential for neurotoxicity, antilice treatment must be used with caution. A child with many open lesions can absorb enough to cause seizures. C Antilice products are not ototoxic. D Products that treat lice are not known to cause bone marrow depression.

Ringworm, frequently found in schoolchildren, is caused by a(n): a. Virus b. Fungus c. Allergic reaction d. Bacterial infection

ANS: B Feedback A These are not the causative organisms for ringworm. B Ringworm is caused by a group of closely related filamentous fungi, which invade primarily the stratum corneum, hair, and nails. They are superficial infections that live on, not in, the skin. C Ringworm is not an allergic response. D These are not the causative organisms for ringworm.

Rocky Mountain spotted fever is caused by the bite of a: a. Flea b. Tick c. Mosquito d. Mouse or rat

ANS: B Feedback A These organisms do not transmit Rocky Mountain spotted fever. B Rocky Mountain spotted fever is caused by a tick. The tick must attach and feed for at least 1 to 2 hours to transmit the disease. The usual habitat of the tick is in heavily wooded areas. C These organisms do not transmit Rocky Mountain spotted fever. D These organisms do not transmit Rocky Mountain spotted fever.

What should the nurse teach an adolescent who is taking tretinoin (Retin-A) to treat acne? a. The medication should be taken with meals. b. Apply sunscreen before going outdoors. c. Wash with benzoyl peroxide before application. d. The effect of the medication should be evident within 1 week.

ANS: B Feedback A Tretinoin is a topical medication. Application is not affected by meals. B Tretinoin causes photosensitivity, and sunscreen should be applied before sun exposure. C If applied together, benzoyl peroxide and tretinoin have reduced effectiveness and a potentially irritant effect. D Optimal results from tretinoin are not achieved for 3 to 5 months.

The primary treatment for warts is: a. Vaccination b. Local destruction c. Corticosteroids d. Specific antibiotic therapy

ANS: B Feedback A Vaccination is prophylaxis for warts and is not a treatment. B Topical treatments include chemical cautery, which is especially useful for the treatment of warts. Local destructive therapy individualized according to location, type, and number. Surgical removal, electrocautery, curettage, cryotherapy, caustic solutions, x-ray treatment, and laser therapies are used. C These are not effective in the treatment of warts. D These are not effective in the treatment of warts.

The nurse is teaching parents of a toddler about language development. Which statement best identifies the characteristics of language development in a toddler? a. Language development skills slow during the toddler period. b. The toddler understands more than he can express. c. Most of the toddler's speech is not easily understood. d. The toddler's vocabulary contains approximately 600 words.

ANS: B Feedback A Although language development varies in relationship to physical activity, language skills are rapidly accelerating by 15 to 24 months of age. B The toddler's ability to understand language (receptive language) exceeds the child's ability to speak it (expressive language). C By 2 years of age, 60% to 70% of the toddler's speech is understandable. D The toddler's vocabulary contains approximately 300 or more words.

The mother of a 10-month-old infant tells the nurse that her infant "really likes cow's milk." What is the nurse's best response to this mother? a. "Milk is good for him." b. "It is best to wait until he is a year old before giving him cow's milk." c. "Limit cow's milk to his bedtime bottle." d. "Mix his cereal with cow's milk and give him formula in a bottle."

ANS: B Feedback A Although milk is a good source of calcium and protein for children after the first year of life, it is not the best source of nutrients for children younger than 1 year old. B It is best to wait until the infant is at least 1 year old before giving him cow's milk because of the risk of allergies and intestinal problems. Cow's milk protein intolerance is the most common food allergy during infancy. C Bedtime bottles of formula or milk are contraindicated because of their high sugar content, which leads to dental decay in primary teeth. D Cereal can be mixed with formula.

An important consideration for the school nurse who is planning a class on bicycle safety is a. Most bicycle injuries involve collision with an automobile. b. Head injuries are the major causes of bicycle-related fatalities. c. Children should wear bicycle helmets if they ride on paved streets. d. Children should not ride double unless the bicycle has an extra-large seat.

ANS: B Feedback A Although motor vehicle collisions do cause injuries to bicyclists, most injuries result from falls. B The most important aspect of bicycle safety is to encourage the rider to use a protective helmet. Head injuries are the major cause of bicycle-related fatalities. C The child should always wear a properly fitted helmet approved by the U.S. Consumer Product Safety Commission. D Children should not ride double.

Approximately how much would a newborn who weighed 7 pounds 6 ounces at birth weigh at 1 year of age? a. 14 3/4 lb b. 22 1/8 lb c. 29 1/2 lb d. Unable to estimate weigh at 1 year

ANS: B Feedback A An infant doubles birth weight by 6 months of age. B An infant triples birth weight by 1 year of age. C An infant quadruples birth weight by 2 years of age. D Weight at 6 months, 1 year, and 2 years of age can be estimated from the birth weight.

Which statement made by a mother is consistent with a developmental delay? a. "I have noticed that my 9-month-old infant responds consistently to the sound of his name." b. "I have noticed that my 12-month-old child does not get herself to a sitting position or pull to stand." c. "I am so happy when my 1 1/2-month-old infant smiles at me." d. "My 5-month-old infant is not rolling over in both directions yet."

ANS: B Feedback A An infant who responds to his name at 9 months of age is demonstrating abilities to both hear and interpret sound. B Critical developmental milestones for gross motor development in a 12-month-old include standing briefly without support, getting to a sitting position, and pulling to stand. If a 12-month-old child does not perform these activities, it may be indicative of a developmental delay. C A social smile is present by 2 months of age. D Rolling over in both directions is not a critical milestone for gross motor development until the child reaches 6 months of age.

What is the best response by the nurse to a parent asking about antidiarrheal medication for her 18-month-old child? a. "It is okay to give antidiarrheal medication to a young child as long as you follow the directions on the box for correct dosage." b. "Antidiarrheal medication is not recommended for young children because it slows the body's attempt to rid itself of the pathogen." c. "I'm sure your child won't like the taste, so give extra fluids when you give the medication." d. "Antidiarrheal medication will lessen the frequency of stools, but give your child Gatorade to maintain electrolyte balance."

ANS: B Feedback A Antidiarrheal medications are not recommended for children younger than 2 years old. B Antidiarrheal medications may actually prolong diarrhea because the body will retain the organism causing the diarrhea, further increasing fluid and electrolyte losses. The use of these medications is not recommended for children younger than 2 years old because of their binding nature and potential for toxicity. C This action is inappropriate because antidiarrheal medications should not be given to a child younger than 2 years old. D It is not appropriate to advise a parent to use antidiarrheal medication for a child younger than 2 years old. Education about appropriate oral replacement fluids includes avoidance of sugary drinks, apple juice, sports beverages, and colas.

Which children are at greater risk for not receiving immunizations? a. Children who attend licensed daycare programs b. Children entering school c. Children who are home schooled d. Young adults entering college

ANS: C Feedback A All states require immunizations for children in daycare programs. B All states require immunizations for children entering school. C Home-schooled children are at risk for being underimmunized and need to be monitored. D Most colleges require a record of immunizations as part of a health history.

Which behavior by parents or teachers will best assist the child in negotiating the developmental task of industry? a. Identifying failures immediately and asking the child's peers for feedback b. Structuring the environment so that the child can master tasks c. Completing homework for children who are having difficulty in completing assignments d. Decreasing expectations to eliminate potential failures

ANS: B Feedback A Asking peers for feedback reinforces the child's feelings of failure. B The task of the caring teacher or parent is to identify areas in which a child is competent and to build on successful experiences to foster feelings of mastery and success. Structuring the environment to enhance self-confidence and to provide the opportunity to solve increasingly more complex problems will promote a sense of mastery. C When teachers or parents complete children's homework for them, it sends the message that you do not trust them to do a good job. Providing assistance and suggestions and praising their best efforts are more appropriate. D Decreasing expectations to eliminate failures will not promote a sense of achievement or mastery.

Which statement is correct about toilet training? a. Bladder training is usually accomplished before bowel training. b. Wanting to please the parent helps motivate the child to use the toilet. c. Watching older siblings use the toilet confuses the child. d. Children must be forced to sit on the toilet when first learning.

ANS: B Feedback A Bowel training precedes bladder training. B Voluntary control of the anal and urethral sphincters is achieved some time after the child is walking. The child must be able to recognize the urge to let go and to hold on. The child must want to please the parent by holding on rather than pleasing himself or herself by letting go. C Watching older siblings provides role modeling and facilitates imitation for the toddler. D The child should be introduced to the potty chair or toilet in a nonthreatening manner.

Which statement by a mother indicates that her 5-month-old infant is ready for solid food? a. "When I give my baby solid foods, she has difficulty getting it to the back of her throat to swallow." b. "She has just started to sit up without any support." c. "I am surprised that she weighs only 11 pounds. I expected her to have gained some weight." d. "I find that she really has to be encouraged to eat."

ANS: B Feedback A Children who are ready to manage solid foods are able to move food to the back of their throats to swallow. This child's extrusion reflex may still be present. B Sitting is a sign that the child is ready to begin with solid foods. C Infants who weigh less than 13 pounds and demonstrate a lack of interest in eating are not ready to be started on solid foods. D Infants who are difficult feeders and do not demonstrate an interest in solid foods are not ready to be started on them.

A 17-year-old tells the nurse that he is not having sex because it would make his parents very angry. This response indicates that the adolescent has a developmental lag in which area? a. Cognitive development b. Moral development c. Psychosocial development d. Psychosexual development

ANS: B Feedback A Cognitive development is related to moral development, but it is not the pivotal point in determining right and wrong behaviors. B The appropriate moral development for a 17-year-old would include evidence that the teenager has internalized a value system and does not depend on parents to determine right and wrong behaviors. Adolescents who remain concrete thinkers may never advance beyond conformity to please others and avoid punishment. C Identity formation is the psychosocial development task. Energy is focused within the adolescent, who exhibits behavior that is self-absorbed and egocentric. D Although a task during adolescence is the development of a sexual identity, the teenager's dependence on the parents' sanctioning of right or wrong behavior is more appropriately related to moral development.

Which statement, made by a 4-year-old child's father, is true about the care of the preschooler's teeth? a. "Because the 'baby teeth' are not permanent, they are not important to the child." b. "My son can be encouraged to brush his teeth after I have thoroughly cleaned his teeth." c. "My son's 'permanent teeth' will begin to come in at 4 to 5 years of age." d. "Fluoride supplements can be discontinued when my son's 'permanent teeth' erupt."

ANS: B Feedback A Deciduous teeth are important because they maintain spacing and play an important role in the growth and development of the jaws and face and in speech development. B Toddlers and preschoolers lack the manual dexterity to remove plaque adequately, so parents must assume this responsibility. C Secondary teeth erupt at approximately 6 years of age. D If the family does not live in an area where fluoride is included in the water supply, fluoride supplements should be continued.

Which is assessed with Tanner staging? a. Hormone levels b. Secondary sex characteristics c. Response to growth hormone secretion tests d. Hyperthyroidism

ANS: B Feedback A Hormone levels are assessed by their concentration in the blood. B Tanner stages are used to assess staging of secondary sex characteristics at puberty. C Growth hormone secretion tests are not associated with Tanner staging. D Tanner stages are not associated with hyperthyroidism.

A mother of a 2-month-old infant tells the nurse, "My child doesn't sleep as much as his older brother did at the same age." What is the best response for the nurse? a. "Have you tried to feed the baby more often?" b. "Infant sleep patterns vary widely, with some infants sleeping only 2 to 3 hours at a time." c. "It is helpful to keep a record of your baby's eating, waking, sleeping, and elimination patterns and to come back in a week to discuss them." d. "This infant is difficult. It is important for you to identify what is bothering the baby."

ANS: B Feedback A Infants typically do not need more caloric intake to improve sleep behaviors. B Newborn infants may sleep as much as 17 to 20 hours per day. Sleep patterns vary widely, with some infants sleeping only 2 to 3 hours at a time. C Keeping intake, output, waking, and sleeping data is not typically helpful to discuss differences among infants' behaviors. D Just because an infant may not sleep as much as a sibling did does not justify labeling the child as being difficult. Identifying an infant as difficult without identifying helpful actions is not a therapeutic response for a parent concerned about sleep.

According to Erikson, the psychosocial task of adolescence is to develop a. Intimacy b. Identity c. Initiative d. Independence

ANS: B Feedback A Intimacy is the developmental stage for early adulthood. B Traditional psychosocial theory holds that the developmental crises of adolescence lead to the formation of a sense of identity. C Initiative is the developmental stage for early childhood. D Independence is not one of Erikson's developmental stages.

What predisposes the adolescent to feel an increased need for sleep? a. An inadequate diet b. Rapid physical growth c. Decreased activity that contributes to a feeling of fatigue d. The lack of ambition typical of this age-group

ANS: B Feedback A Rapid physical growth, the tendency toward overexertion, and the overall increased activity of this age contributes to fatigue. B During growth spurts, the need for sleep is increased. C Rapid physical growth, the tendency toward overexertion, and the overall increased activity of this age contributes to fatigue. D Rapid physical growth, the tendency toward overexertion, and the overall increased activity of this age contributes to fatigue.

Which statement is the most appropriate advice to give parents of a 16-year-old who is rebellious? a. "You need to be stricter so that your teen stops trying to test the limits." b. "You need to collaborate with your daughter and set limits that are perceived as being reasonable." c. "Increasing your teen's involvement with her peers will improve her self-esteem." d. "Allow your teenager to choose the type of discipline that is used in your home."

ANS: B Feedback A Setting stricter limits typically does not decrease rebelliousness or decrease testing of parental limits. B Allowing teenagers to choose between realistic options and offering consistent and structured discipline typically enhances cooperation and decreases rebelliousness. Structure helps adolescents to feel more secure and assists them in the decision-making process. C Increasing peer involvement does not typically increase self-esteem. D Allowing teenagers to choose the method of discipline is not realistic and typically does not reduce rebelliousness.

Many adolescents decide to follow a vegetarian diet during their teen years. The nurse can advise the adolescent and his or her parents that a. This diet will not meet the nutritional requirements of growing teens. b. A vegetarian diet is healthy for this population. c. An adolescent on a vegetarian diet is less likely to eat high-fat or low-nutrient foods. d. A vegetarian diet requires little extra meal planning.

ANS: B Feedback A Several dietary organizations have suggested that a vegetarian diet, if correctly followed, is healthy for this population. B A vegetarian diet is healthy for this population, and the low-fat aspect of the diet can prevent future cardiovascular problems. C As with any adolescent, nurses need to advise teens who follow a vegetarian eating plan to avoid low-nutrient, high-fat foods. D The nurse can assist with planning food choices that will provide sufficient calories and necessary nutrients. The focus is on obtaining enough calories for growth and energy from a variety of fruits and vegetables, whole grains, nuts, and soymilk.

A nurse is assessing an older school-age child recently admitted to the hospital. Which assessment indicates that the child is in an appropriate stage of cognitive development? a. The child's addition and subtraction ability b. The child's ability to classify c. The child's vocabulary d. The child's play activity

ANS: B Feedback A Subtraction and addition are appropriate cognitive activities for the young school-age child. B The ability to classify things from simple to complex and the ability to identify differences and similarities are cognitive skills of the older school-age child; this demonstrates use of classification and logical thought processes. C Vocabulary is not as valid an assessment of cognitive ability as is the child's ability to classify. D Play activity is not as valid an assessment of cognitive function as is the child's ability to classify.

What is the most important factor in determining the rate of fluid replacement in the dehydrated child? a. The child's weight b. The type of dehydration c. Urine output d. Serum potassium level

ANS: B Feedback A The child's weight determines the amount of fluid needed, not the rate of fluid replacement. One milliliter of body fluid is equal to 1 g of body weight; therefore a loss of 1 kg (2.2 lb) is equal to 1 L of fluid. B Isonamtremic and hyponatremic dehydration resuscitation involves fluid replacement over 24 hours. Hypernatremic dehydration involves a slower replacement rate to prevent a sudden decrease in the sodium level. C Urine output is not a consideration for determining the rate of administration of replacement fluids. D Potassium level is not as significant in determining the rate of fluid replacement as the type of dehydration.

Which behavior suggests appropriate psychosocial development in the adolescent? a. The adolescent seeks validation for socially acceptable behavior from older adults. b. The adolescent is self-absorbed and self-centered and has sudden mood swings. c. Adolescents move from peers and enjoy spending time with family members. d. Conformity with the peer group increases in late adolescence.

ANS: B Feedback A The peer group validates acceptable behavior during adolescence. B During adolescence, energy is focused within. Adolescents concentrate on themselves in an effort to determine who they are or who they will be. Adolescents are likely to be impulsive and impatient. Parents often describe their teenager as being "self-centered or lazy." C Adolescents move from family and enjoy spending time with peers. Adolescents also spend time alone; they need this time to think and concentrate on themselves. D Conformity becomes less important in late adolescence.

Which statement by a mother of a toddler indicates a correct understanding of the use of discipline? a. "I always include explanations and morals when I am disciplining my toddler." b. "I always try to be consistent when disciplining the children, and I correct my children at the time they are misbehaving." c. "I believe that discipline should be done by only one family member." d. "My rule of thumb is no more than one spanking a day."

ANS: B Feedback A The toddler's cognitive level of development precludes the use of explanations and morals as a part of discipline. B Consistent and immediate discipline for toddlers is the most effective approach. Unless disciplined immediately, the toddler will have difficulty connecting the discipline with the behavior. C Discipline for the toddler should be immediate; therefore the family member caring for the child should provide discipline to the toddler when it is necessary. D Discipline is required for unacceptable behavior, and the one-spanking-a-day rule contradicts the concept of a consistent response to inappropriate behavior. In addition, spanking is an inappropriate method of disciplining a child.

Which is an appropriate disciplinary intervention for the school-age child? a. Using time-out periods b. Using a consequence that is consistent with the inappropriate behavior c. Using physical punishment d. Using lengthy dialog about inappropriate behavior

ANS: B Feedback A Time-out periods are more appropriate for younger children. B A consequence that is related to the inappropriate behavior is the recommended discipline. C Physical intervention is an inappropriate form of discipline. It does not connect the discipline with the child's inappropriate behavior. D Lengthy discussions typically are not helpful.

The mother of a 14-month-old child is concerned because the child's appetite has decreased. The best response for the nurse to make to the mother is a. "It is important for your toddler to eat three meals a day and nothing in between." b. "It is not unusual for toddlers to eat less." c. "Be sure to increase your child's milk consumption, which will improve nutrition." d. "Giving your child a multivitamin supplement daily will increase your toddler's appetite."

ANS: B Feedback A Toddlers need small, frequent meals. Nutritious selection throughout the day, rather than quantity, is more important with this age-group. B Physiologically, growth slows and appetite decreases during the toddler period. C Milk consumption should not exceed 16 to 24 oz daily. Juice should be limited to 4 to 6 oz per day. Increasing the amount of milk will only further decrease solid food intake. D Supplemental vitamins are important for all children, but they do not increase appetite.

A mother asks when toilet training is most appropriately initiated. What is the nurse's best response? a. "When your child is 12 to 18 months of age." b. "When your child exhibits signs of physical and psychological readiness." c. "When your child has been walking for 9 months." d. "When your child is able to sit on the 'potty' for 10 to 15 minutes."

ANS: B Feedback A Toilet training is not arbitrarily started at 12 to 18 months of age. The child needs to demonstrate signs of bowel or bladder control before attempting toilet training. The average toddler is not ready until 18 to 24 months of age. Waiting until 24 to 30 months of age makes the task easier; toddlers are less negative, more willing to control their sphincters, and want to please their parents. B Neurologic development is completed at approximately 18 months of age. Parents need to know that both physical and psychological readiness are necessary for toilet training to be successful. C One of the physical signs of readiness for toilet training is that the child has been walking for 1 year. D The ability to sit on the "potty" for 10 to 15 minutes may demonstrate parental control rather than being a sign of developmental readiness for toilet training.

Tattoos have become increasingly popular among mainstream adolescents. Like clothing and hairstyles, tattoos serve to define one's identity. It is important for nurses to caution adolescents on the health risks of obtaining a tattoo. These include (select all that apply) a. Amateur tattoos are difficult to remove. b. Tattoos pose a risk for bloodborne and skin infections. c. Health care professionals must be notified of the existence of a tattoo before a magnetic resonance imaging (MRI) scan. d. Tattoo dyes may cause allergic reactions. e. Tattoo parlors are well regulated.

ANS: B, C, D Feedback Correct Tattoos carry the risk for contracting bloodborne diseases such as hepatitis B and HIV. Infection, allergic reaction to the dye, scarring, or keloid formation can occur. Should an MRI ever be required, it is important to notify the health care professionals, because the dyes can contain iron and other metals. Incorrect Amateur tattoos are easily removed; however, studio tattoos made with red and green dye are extremely difficult to remove. Very little regulation exists in the tattoo industry; therefore, the cleanliness of each tattoo parlor varies. Teens should be counseled to avoid making an impulsive decision to get a tattoo.

A nurse is planning care for a hospitalized toddler in the preoperational thinking stage. Which characteristics should the nurse expect in this stage? Select all that apply. a. Concrete thinking b. Egocentrism c. Animism d. Magical thought e. Ability to reason

ANS: B, C, D Feedback Correct The characteristics of preoperational thinking that occur for the toddler include egocentrism (views everything in relation to self), animism (believes that inert objects are alive), and magical thought (believes that thinking something causes that event). Incorrect Concrete thinking is seen in school-age children, and ability to reason is seen with adolescents.

The prevalence of obesity in the United States has risen dramatically in both adults and children. The increase in the number of overweight children is addressed in Healthy People 2020. Strategies designed to approach this issue include (select all that apply) a. Decreased calcium and iron intake b. Increased fiber and whole grain intake c. Decreased use of sugar and sodium d. Increase fruit and vegetable intake e. Decrease the use of solid fats

ANS: B, C, D, E Feedback Correct Along with these recommendations, children at risk for being overweight should be screened beginning at age 2 years. Children with a family history of dyslipidemia or early cardiovascular disease development, children whose body mass index percentile exceeds the definition for overweight, and children who have high blood pressure should have a fasting lipid screen. Incorrect The nurse should instruct parents that calcium and iron intake should be increased as part of this strategy.

Which strategies can a nurse teach to parents of a child experiencing uncomplicated school refusal? Select all that apply. a. The child should be allowed to stay home until the anxiety about going to school is resolved. b. Parents should be empathetic yet firm in their insistence that the child attends school. c. A modified school attendance may be necessary. d. Parents need to pick the child up at school whenever the child wants to come home. e. Parents need to communicate with the teachers about the situation.

ANS: B, C, E Feedback Correct In uncomplicated cases of school refusal, the parent needs to return the child to school as soon as possible. If symptoms are severe, a limited period of part-time or modified school attendance may be necessary. For example, part of the day may be spent in the counselor's or school nurse's office, with assignments obtained from the teacher. Parents should be empathetic yet firm and consistent in their insistence that the child attend school. Incorrect Parents should not pick the child up at school once the child is there. The principal and teacher should be told about the situation so that they can cooperate with the treatment plan.

Parents of a teenager ask the nurse what signs they should look for if their child is in a gang. The nurse should include which signs when answering? Select all that apply. a. Plans to try out for the debate team at school b. Skipping classes to go to the mall c. Hanging out with friends they have had since childhood d. Unexplained source of money e. Fear of the police

ANS: B, D, E Feedback Correct Signs of gang involvement include skipping classes, unexplained sources of money, and fear of the police. Associating with new friends while ignoring old friends is also a sign. A change in attitude toward participating in activities is another sign of gang involvement. Incorrect Plans to try out for the debate team at school are not a sign of gang involvement. Hanging out with friends he or she has had since childhood is not a sign of gang involvement.

Which factor has the greatest influence on child growth and development? a. Culture b. Environment c. Genetics d. Nutrition

ANS: C Feedback A Culture is a significant factor that influences how children grow toward adulthood. Culture influences both growth and development but does not eliminate inborn genetic influences. B Environment has a significant role in determining growth and development both before and after birth. The environment can influence how and to which extent genetic traits are manifested, but environmental factors cannot eliminate the effect of genetics. C Genetic factors (heredity) determine each individual's growth and developmental rate. Although factors such as environment, culture, nutrition, and family can influence genetic traits, they do not eliminate the effect of the genetic endowment, which is permanent. D Nutrition is critical for growth and plays a significant role throughout childhood.

Frequent developmental assessments are important for which reason? a. Stable developmental periods during infancy provide an opportunity to identify any delays or deficits. b. Infants need stimulation specific to the stage of development. c. Critical periods of development occur during childhood. d. Child development is unpredictable and needs monitoring.

ANS: C Feedback A Infancy is a dynamic time of development that requires frequent evaluations to assess appropriate developmental progress. B Infants in a nurturing environment will develop appropriately and will not necessarily need stimulation specific to their developmental stage. C Critical periods are blocks of time during which children are ready to master specific developmental tasks. The earlier those delays in development are discovered and intervention initiated, the less dramatic their effect will be. D Normal growth and development is orderly and proceeds in a predictable pattern based on each individual's abilities and potentials.

A 2-month-old child has not received any immunizations. Which immunizations should the nurse give? a. DTaP, Hib, HepB, MCV, varicella b. DTaP, Hib, HepB, HPV, IPV, Rota c. DTaP, Hib, HepB, PCV, Rota d. DTaP, Hib, HepB, PCV, HepA

ANS: C Feedback A Meningococcal vaccine should be administered to children at 11 to 12 years of age. B Human papillomavirus vaccine is administered to adolescent girls only. C DTaP, Hib, HepB, PCV, IPV, and Rota are appropriate immunizations for an unimmunized 2-month-old child. D HepA is recommended for all children at 1 year of age.

What does the nurse need to know when observing a chronically ill child at play? a. Play is not important to hospitalized children. b. Children need to have structured play periods. c. Children's play is a form of communication. d. Play is to be discouraged because it tires hospitalized children.

ANS: C Feedback A Play is important to all children in all environments. Play for children is a mechanism for mastering their environment. B Although children's play activities appear to be unorganized and at times chaotic, play has purpose and meaning. Imposing structure on play interferes with the tasks being worked on. C Play for all children is an activity woven with meaning and purpose. For chronically ill children, play can indicate their state of wellness and response to treatment. It is a way to express joy, fear, anxiety, and disappointments. D Children who have fewer energy reserves still require play. For these children, less-active play activities will be important.

The nurse is planning a teaching session for a young child and her parents. According to Piaget's theory, the period of cognitive development in which the child is able to distinguish between concepts related to fact and fantasy, such as human beings are incapable of flying like birds, is the _______ period of cognitive development. a. Sensorimotor b. Formal operations c. Concrete operations d. Preoperational

ANS: C Feedback A The sensorimotor stage occurs in infancy and is a period of reflexive behavior. During this period, the infant's world becomes more permanent and organized. The stage ends with the infant demonstrating some evidence of reasoning. B Formal operations is a period in development in which new ideas are created through previous thoughts. Analytic reason and abstract thought emerge in this period. C Concrete operations is the period of cognitive development in which children's thinking is shifted from egocentric to being able to see another's point of view. They develop the ability to distinguish fact from fantasy. D The preoperational stage is a period of egocentrism in which the child's judgments are illogical and dominated by magical thinking and animism.

A mother calls the emergency department nurse because her child was stung by a scorpion. The nurse should recommend: a. Administering antihistamine b. Cleansing with soap and water c. Keeping child quiet and come to emergency department d. Removing stinger and apply cool compresses

ANS: C Feedback A Antihistamines are not effective against scorpion venom. B The wound will have intense local pain. Emergency treatment is indicated. C Venomous species of scorpions inject venom that contains hemolysins, endotheliolysins, and neurotoxins. The absorption of the venom is delayed by keeping the child quiet and the involved area in dependent position. D The wound will have intense local pain. Emergency treatment is indicated.

Treatment for herpes simplex virus (types 1 or 2) includes: a. Corticosteroids b. Oral griseofulvin c. Oral antiviral agent d. Topical and/or systemic antibiotic

ANS: C Feedback A Corticosteroids are not effective for viral infections. B Griseofulvin is an antifungal agent and not effective for viral infections. C Oral antiviral agents are effective for viral infections such as herpes simplex. D Antibiotics are not effective in viral diseases.

An important nursing consideration when caring for a child with impetigo contagiosa is to: a. Apply topical corticosteroids to decrease inflammation. b. Carefully remove dressings so as not to dislodge undermined skin, crusts, and debris. c. Carefully wash hands and maintain cleanliness when caring for an infected child. d. Examine child under a Wood lamp for possible spread of lesions.

ANS: C Feedback A Corticosteroids are not indicated in bacterial infections. B Dressings are usually not indicated. The undermined skin, crusts, and debris are carefully removed after softening with moist compresses. C A major nursing consideration related to bacterial skin infections, such as impetigo contagiosa, is to prevent the spread of the infection and complications. This is done by thorough handwashing before and after contact with the affected child. D A Wood lamp is used to detect fluorescent materials in the skin and hair. It is used in certain disease states, such as tinea capitis.

To assess the child with severe burns for adequate perfusion, the nurse monitors: a. Distal pulses b. Skin turgor c. Urine output d. Mucous membranes

ANS: C Feedback A Distal pulses may be affected by many variables. Urine output is the most reliable indicator of end-organ perfusion. B Skin turgor is often difficult to assess on burn patients because the skin is not intact. C Urine output reflects the adequacy of end-organ perfusion. D Mucous membranes do not reflect end-organ perfusion.

What best describes a full-thickness (third-degree) burn? a. Erythema and pain b. Skin showing erythema followed by blister formation c. Destruction of all layers of skin evident with extension into subcutaneous tissue d. Destruction injury involving underlying structures such as muscle, fascia, and bone

ANS: C Feedback A Erythema and pain are characteristic of a first-degree burn or superficial burn. B Erythema with blister formation is characteristic of a second-degree or partial-thickness burn. C A third-degree or full-thickness burn is a serious injury that involves the entire epidermis and dermis and extends into the subcutaneous tissues. D A fourth-degree burn is a full-thickness burn that also involves underlying structures such as muscle, fascia, and bone.

With what beverage should the parents of a child with ringworm be taught to give griseofulvin? a. Water b. A carbonated drink c. Milk d. Fruit juice

ANS: C Feedback A Griseofulvin is insoluble in water. B Carbonated drinks do not contain fat, which aids in the absorption of griseofulvin. C Griseofulvin is insoluble in water. Giving the medication with a high-fat meal or milk increases absorption. D Fruit juice does not contain any fat; fat aids absorption of the medication.

A father calls the clinic nurse because his 2-year-old child was bitten by a black widow spider. The nurse should advise the father to: a. Apply warm compresses. b. Carefully scrape off stinger. c. Take child to emergency department. d. Apply a thin layer of corticosteroid cream.

ANS: C Feedback A Warm compresses increase the circulation to the area and facilitate the spread of the venom. B The black widow spider does not have a stinger. C The black widow spider has a venom that is toxic enough to be harmful. The father should take the child to the emergency department for immediate treatment. D Corticosteroid cream will have no effect on the venom.

Which milestone is developmentally appropriate for a 2-month-old infant? a. Pulled to a sitting position, head lag is absent. b. Pulled to a sitting position, the infant is able to support the head when the trunk is lifted. c. The infant can lift his or her head from the prone position and briefly hold the head erect. d. In the prone position, the infant is fully able to support and hold the head in a straight line.

ANS: C Feedback A A 2-month-old infant's neck muscles are stronger than those of a newborn; however, head lag is present when pulled to a sitting position. B A 2-month-old infant continues to have some head lag when pulled to a sitting position. C A 2-month-old infant is able to briefly hold the head erect when in a prone position. If a parent were holding the infant against the parent's shoulder, the infant would be able to lift his or her head briefly. D It is not until 4 months of age that the infant can easily lift his or her head and hold it steadily erect when in the prone position.

Which statement is the most accurate about moral development in the 9-year-old school-age child? a. Right and wrong are based on physical consequences of behavior. b. The child obeys parents because of fear of punishment. c. The school-age child conforms to rules to please others. d. Parents are the determiners of right and wrong for the school-age child.

ANS: C Feedback A Children 4 to 7 years of age base right and wrong on consequences. B Consequences are the most important consideration for the child between 4 and 7 years of age. C The 7- to 12-year-old child bases right and wrong on a good-boy or good-girl orientation in which the child conforms to rules to please others and avoid disapproval. D Parents determine right and wrong for the child younger than 4 years of age.

Which assessment is most relevant to the care of an infant with dehydration? a. Temperature, heart rate, and blood pressure. b. Respiratory rate, oxygen saturation, and lung sounds. c. Heart rate, sensorium, and skin color. d. Diet tolerance, bowel function, and abdominal girth.

ANS: C Feedback A Children can compensate and maintain an adequate cardiac output when they are hypovolemic. Blood pressure is not as reliable an indicator of shock as are changes in heart rate, sensorium, and skin color. B Respiratory assessments will not provide data about impending hypovolemic shock. C Changes in heart rate, sensorium, and skin color are early indicators of impending shock in the child. D Diet tolerance, bowel function, and abdominal girth are not as important indicators of shock as heart rate, sensorium, and skin color.

A mother tells the nurse that she is discontinuing breastfeeding her 5-month-old infant. The nurse should recommend that the infant be given a. Skim milk b. Whole cow's milk c. Commercial iron-fortified formula d. Commercial formula without iron

ANS: C Feedback A Cow's milk should not be used in children younger than 12 months. B Cow's milk should not be used in children younger than 12 months. C For children younger than 1 year, the American Academy of Pediatrics recommends the use of breast milk. If breastfeeding has been discontinued, then iron-fortified commercial formula should be used. D Maternal iron stores are almost depleted by this age; the iron-fortified formula will help prevent the development of iron-deficiency anemia.

What is the primary purpose of a transitional object? a. It helps the parents deal with the guilt they feel when they leave the child. b. It keeps the child quiet at bedtime. c. It is effective in decreasing anxiety in the toddler. d. It decreases negativism and tantrums in the toddler.

ANS: C Feedback A Decreased parental guilt (distress) is an indirect benefit of a transitional object. B A transitional object may be part of a bedtime ritual, but it may not keep the child quiet at bedtime. C Decreasing anxiety, particularly separation anxiety, is the function of a transitional object; it provides comfort to the toddler in stressful situations and helps make the transition from dependence to autonomy. D A transitional object does not significantly affect negativity and tantrums, but it can comfort a child after tantrums.

Why do peer relationships become more important during adolescence? a. Adolescents dislike their parents. b. Adolescents no longer need parental control. c. They provide adolescents with a feeling of belonging. d. They promote a sense of individuality in adolescents.

ANS: C Feedback A During adolescence, the parent/child relationship changes from one of protection-dependency to one of mutual affection and quality. B Parents continue to play an important role in the personal and health-related decisions. C The peer group serves as a strong support to teenagers, providing them with a sense of belonging and a sense of strength and power. D The peer group forms the transitional world between dependence and autonomy.

What assessment should the nurse make before initiating an intravenous (IV) infusion of dextrose 5% in 0.9% normal saline solution with 10 mEq of potassium chloride for a child hospitalized with dehydration? a. Fluid intake b. Number of stools c. Urine output d. Capillary refill

ANS: C Feedback A Fluid intake does not give information about renal function. B Stool count sheds light on intestinal function. Renal function is the concern before potassium chloride is added to an IV solution. C Potassium chloride should never be added to an IV solution in the presence of oliguria or anuria (urine output less than 0.5 mL/kg/hr). D Assessment of capillary refill does not provide data about renal function.

At what age is an infant first expected to locate an object hidden from view? a. 4 months of age b. 6 months of age c. 9 months of age d. 20 months of age

ANS: C Feedback A Four-month-old infants are not cognitively capable of searching out objects hidden from their view. Infants at this developmental level do not pursue hidden objects. B Six-month-old infants have not developed the ability to perceive objects as permanent and do not search out objects hidden from their view. C By 9 months of age, an infant will actively search for an object that is out of sight. D Twenty-month-old infants actively pursue objects not in their view and are capable of recalling the location of an object not in their view. They first look for hidden objects around age 9 months.

In general, the earliest age at which puberty begins is ____ years in girls and _____ years in boys. a. 13; 13 b. 11; 11 c. 10; 12 d. 12; 10

ANS: C Feedback A Girls and boys do not usually begin puberty at the same age; girls usually begin earlier than boys. B Girls and boys do not usually begin puberty at the same age; girls usually begin earlier than boys. C Puberty signals the beginning of the development of secondary sex characteristics. This begins in girls earlier than in boys. Usually, there is a 2-year difference in the age at onset. D Girls and boys do not usually begin puberty at the same age; girls usually begin earlier than boys.

Which statement by the nurse is most appropriate to a 15-year-old whose friend has mentioned suicide? a. "Tell your friend to come to the clinic immediately." b. "You need to gather details about your friend's suicide plan." c. "Your friend's threat needs to be taken seriously, and immediate help for your friend is important." d. "If your friend mentions suicide a second time, you will want to get your friend some help."

ANS: C Feedback A Instructing a 15-year-old to tell a friend to come to the clinic immediately provides the teen with limited information and does not address the concern. B It is important to determine whether a person threatening suicide has a plan of action; however, the best information for the 15-year-old to have is that all threats of suicide should be taken seriously and immediate help is important. C Suicide is the third most common cause of death among American adolescents. A suicide threat from an adolescent serves as a dramatic message to others and should be taken seriously. Adolescents at risk should be targeted for supportive guidance and counseling before a crisis occurs. D It is imperative that help is provided immediately for a teenager who is talking about suicide. Waiting until the teen discusses it a second time may be too late.

The parents of a newborn infant state, "We will probably not have our baby immunized because we are concerned about the risk of our child being injured." What is the nurse's best response? a. "It is your decision." b. "Have you talked with your parents about this? They can probably help you think about this decision." c. "The risks of not immunizing your baby are greater than the risks from the immunizations." d. "You are making a mistake."

ANS: C Feedback A It is the parents' decision not to immunize the child; however, the nurse has a responsibility to inform parents about the risks to infants who are not immunized. B Grandparents can be supportive but are not the primary decision makers for the infant. C Although immunizations have been documented to have a negative effect in a small number of cases, an unimmunized infant is at greater risk for development of complications from childhood diseases than from the vaccines. D Telling parents that they are making a mistake is an inappropriate response.

A group of boys ages 9 and 10 years have formed a "boys-only" club that is open to neighborhood and school friends who have skateboards. This should be interpreted as a. Behavior that encourages bullying and sexism b. Behavior that reinforces poor peer relationships c. Characteristic of social development of this age d. Characteristic of children who later are at risk for membership in gangs

ANS: C Feedback A Peer-group identification and association are essential to a child's socialization. Poor relationships with peers and a lack of group identification can contribute to bullying. B Peer-group identification and association are essential to a child's socialization. Poor relationships with peers and a lack of group identification can contribute to bullying. C One of the outstanding characteristics of middle childhood is the creation of formalized groups or clubs. D A boys-only club does not have a direct correlation with later gang activity.

The parents of a 14-year-old girl are concerned that their adolescent spends too much time looking in the mirror. Which statement is the most appropriate for the nurse to make? a. "Your teenager needs clearer and stricter limits about her behavior." b. "Your teenager needs more responsibility at home." c. "During adolescence this behavior is not unusual." d. "The behavior is abnormal and needs further investigation."

ANS: C Feedback A Stricter limits are not an appropriate response for a behavior that is part of normal development. B More responsibility at home is not an appropriate response for this situation. C Egocentric and narcissistic behavior is normal during this period of development. The teenager is seeking a personal identity. D The behavior is normal and needs no further investigation.

The nurse is explaining Tanner staging to an adolescent and her mother. Which statement best describes Tanner staging? a. Predictable stages of puberty that are based on chronologic age b. Staging of puberty based on the initiation of menarche and nocturnal emissions c. Predictable stages of puberty that are based on primary and secondary sexual characteristics d. Staging of puberty based on the initiation of primary sexual characteristics

ANS: C Feedback A Tanner stages are not based on chronologic age. The age at which an adolescent enters puberty is variable. B The puberty stage in girls begins with breast development. Puberty stage in boys begins with genital enlargement. C Tanner sexual-maturing ratings are based on the development of stages of primary and secondary sexual characteristics. D Primary sexual characteristics are not the basis of Tanner staging.

In providing anticipatory guidance to parents whose child will soon be entering kindergarten, which is a critical factor in preparing a child for kindergarten entry? a. The child's ability to sit still b. The child's sense of learned helplessness c. The parent's interactions and responsiveness to the child d. Attending a preschool program

ANS: C Feedback A The child's ability to sit still is important to learning; however, parental responsiveness and involvement are more important factors. B Learned helplessness is the result of a child feeling that he or she has no effect on the environment and that his or her actions do not matter. Parents who are actively involved in a supportive learning environment will demonstrate a more positive approach to learning. C Interactions between the parent and child are an important factor in the development of academic competence. Parental encouragement and support maximize a child's potential. D Preschool and daycare programs can supplement the developmental opportunities provided by parents at home, but they are not critical in preparing a child for entering kindergarten.

Which is a priority in counseling parents of a 6-month-old infant? a. Increased appetite from secondary growth spurt b. Encouraging the infant to smile c. Securing a developmentally safe environment for the infant d. Strategies to teach infants to sit up

ANS: C Feedback A The infant's appetite and growth velocity decrease in the second half of infancy. B Although a social smile should be present by 6 months of age, encouraging this is not of higher priority than ensuring environmental safety. C Safety is a primary concern as an infant becomes increasingly mobile. D Unless the infant has a neuromuscular deficit, strategies for teaching a normally developing infant to sit up are not necessary.

The parent of 2-week-old Sarah asks the nurse whether Sarah needs fluoride supplements, because she is exclusively breastfed. The nurse's best response is a. "She needs to begin taking them now." b. "They are not needed if you drink fluoridated water." c. "She may need to begin taking them at age 6 months." d. "She can have infant cereal mixed with fluoridated water instead of supplements."

ANS: C Feedback A The recommendation is to begin supplementation at 6 months. B The amount of water that is ingested and the amount of fluoride in the water are considered when supplementation is being considered. C Fluoride supplementation is recommended by the American Academy of Pediatrics beginning at age 6 months if the child is not drinking adequate amounts of fluoridated water. D The amount of water that is ingested and the amount of fluoride in the water are considered when supplementation is being considered.

According to Piaget, the 6-month-old infant is in what stage of the sensorimotor phase? a. Use of reflexes b. Primary circular reactions c. Secondary circular reactions d. Coordination of secondary schemata

ANS: C Feedback A The use of reflexes is primarily during the first month of life. B Primary circular reaction stage marks the replacement of reflexes with voluntary acts. The infant is in this stage from age 1 month to 4 months. C Infants are usually in the secondary circular reaction stage from age 4 months to 8 months. This stage is characterized by a continuation of the primary circular reaction because of the response that results. Shaking is performed to hear the noise of the rattle, not just for shaking. D The fourth sensorimotor stage is coordination of secondary schemata. This is a transitional stage in which increasing motor skills enable greater exploration of the environment.

Which comments indicate that the mother of a toddler needs further teaching about dental care? a. "We use well water so I give my toddler fluoride supplements." b. "My toddler brushes his teeth with my help." c. "My child will not need a dental checkup until his permanent teeth come in." d. "I use a small nylon bristle brush for my toddler's teeth."

ANS: C Feedback A Toddlers need fluoride supplements when they use a water supply that is not fluorinated. B Toddlers need supervision with dental care. The parent should finish brushing areas not reached by the child. C Children should first see the dentist 6 months after the first primary tooth erupts and no later than age 30 months. D A small nylon bristle brush works best for cleaning toddlers' teeth.

A nurse is instructing parents on treatment of pediculosis (head lice). Which should the nurse include in the teaching plan? (Select all that apply.): a. Bedding should be washed in warm water and dried on a low setting. b. After treating the hair and scalp with a pediculicide, shampoo the hair with regular shampoo. c. Retreat the hair and scalp with a pediculicide in 7 to 10 days. d. Items that cannot be washed should be dry cleaned or sealed in plastic bags for 2 to 3 weeks. e. Combs and brushes should be boiled in water for at least 10 minutes.

ANS: C, D, E Feedback Correct: An over-the-counter pediculicide, permethrin 1% (Nix, Elimite, Acticin), kills head lice and eggs with one application and has residual activity (i.e., it stays in the hair after treatment) for 10 days. Nix crème rinse is applied to the hair after it is washed with a conditioner-free shampoo. The product should be rinsed out after 10 minutes. Incorrect: The hair should not be shampooed for 24 hours after the treatment. Even though the kill rate is high and there is residual action, retreatment should occur after 7 to 10 days. Combs and brushes should be boiled or soaked in antilice shampoo or hot water (greater than 60° C [140° F]) for at least 10 minutes. Advise parents to wash clothing (especially hats and jackets), bedding, and linens in hot water and dry at a hot dryer setting.

What should the nurse evaluate before administering the Denver Developmental Screening Test II (DDST-II)? Select all that apply. a. The child's height and weight b. The parent's ability to comprehend the results c. The child's mood d. The parent-child interaction e. The child's chronologic age

ANS: C, E Feedback Correct The results of the screening test are valid if the child acted in a normal and expected manner. The child's chronologic age in years, months, and days must be calculated in order to draw the age line. This is necessary in order to perform an accurate DDST-II. Reliability and validity of the test can be altered if the child is not feeling well or is under the influence of medications. Incorrect The child's height and weight are not relevant to the DDST-II screening process. The parent's ability to understand the results of the screening is not relevant to the validity of the test. The parent-child interaction is not significantly relevant to the test results.

When counseling parents and children about the importance of increased physical activity, the nurse can emphasize a. Anaerobic exercise should comprise a major component of the child's daily exercise. b. All children should be physically active for at least 2 hours per day. c. It is not necessary to participate in physical education classes at school if a student is taking part in other activities. d. Making exercise fun and a habitual activity.

ANS: D Feedback A Aerobic exercise should comprise a major component of children's daily exercise; however, physical activity should also include muscle and bone strengthening activities. B Children and adolescents should be physically active for at least 1 hour daily. C Encourage all student to participate fully in any physical education classes. D It is important to make exercise a fun and a habitual activity. Encourage parents to investigate their community's different activity programs. This includes recreation centers, parks, and the YMCA.

Which immunizations should be used with caution in children with an allergy to eggs? a. HepB b. DTaP c. Hib d. MMR

ANS: D Feedback A HepB is safe for children with an egg allergy. B DTaP is safe for children with an egg allergy. C Hib is safe for children with an egg allergy. D Live measles vaccine is produced by using chick embryo cell culture, so there is a remote possibility of anaphylactic hypersensitivity in children with egg allergies. Most reactions are actually the result of other components in the vaccine.

What procedure is contraindicated in the care of a child with a minor partial-thickness burn injury wound? a. Cleaning the affected area with mild soap and water b. Applying antimicrobial ointment to the burn wound c. Changing dressings daily d. Leaving all loose tissue or skin intact

ANS: D Feedback A Cleaning with mild soap and water are important to the healing process. B Antimicrobial ointment is used on the burn wound to fight infection. C Clean dressings are applied daily to prevent wound infection. When dressings are changed, the condition of the burn wound can be assessed. D All loose skin and tissue should be debrided, because it can become a breeding ground for infectious organisms.

The pediatric nurse understands that cellulitis is most often caused by: a. Herpes zoster b. Candida albicans c. Human papillomavirus d. Streptococcus or Staphylococcus organisms

ANS: D Feedback A Herpes zoster is the virus associated with varicella and shingles. B Candida albicans is associated with candidiasis or thrush. C Human papillomavirus is associated with various types of human warts. D Streptococcus, Staphylococcus, and Haemophilus influenzae are the organisms usually responsible for cellulitis

The skin condition commonly known as "warts" is the result of an infection by which organism? a. Bacteria b. Fungus c. Parasite d. Virus

ANS: D Feedback A Infection with these organisms does not result in warts. B Infection with these organisms does not result in warts. C Infection with these organisms does not result in warts. D Human warts are caused by the human papillomavirus.

What should the parents of an infant with thrush (oral candidiasis) be taught about medication administration? a. Give nystatin suspension with a syringe without a needle. b. Apply nystatin cream to the affected area twice a day. c. Give nystatin before the infant is fed. d. Swab nystatin suspension onto the oral mucous membranes after feedings.

ANS: D Feedback A Medication may not reach the affected areas when it is squirted into the infant's mouth. Rubbing the suspension onto the gum ensures contact with the affected areas. B Nystatin cream is used for diaper rash caused by Candida. C To prolong contact with the affected areas, the medication should be administered after a feeding. D It is important to apply the nystatin suspension to the affected areas, which is best accomplished by rubbing it onto the gums and tongue, after feedings, every 6 hours, until 3 to 4 days after symptoms have disappeared.

When taking a history on a child with a possible diagnosis of cellulitis, what should be the priority nursing assessment to help establish a diagnosis? a. Any pain the child is experiencing b. Enlarged, mobile, and nontender lymph nodes c. Child's urinalysis results d. Recent infections or signs of infection

ANS: D Feedback A Pain is important, but the history of recent infections is more relevant to the diagnosis. B Lymph nodes may be enlarged (lymphadenitis), but they are not mobile and are nontender. Lymphangitis may be seen with red "streaking" of the surrounding area. C An abnormal urinalysis result is not usually associated with cellulitis. D Cellulitis may follow an upper respiratory infection, sinusitis, otitis media, or a tooth abscess. The affected area is red, hot, tender, and indurated.

The process of burn shock continues until what physiologic mechanism occurs? a. Heart rate returns to normal. b. Airway swelling decreases. c. Body temperature regulation returns to normal. d. Capillaries regain their seal.

ANS: D Feedback A The heart rate will be increased throughout the healing process because of increased metabolism. B Airway swelling subsides over a period of 2 to 5 days after injury. C Body temperature regulation will not be normal until healing is well under way. D Within minutes of the burn injury, the capillary seals are lost with a massive fluid leakage into the surrounding tissue, resulting in burn shock. The process of burn shock continues for approximately 24 to 48 hours, when capillary seals are restored.

Which statement made by a parent indicates an understanding about the management of a child with cellulitis? a. "I am supposed to continue the antibiotic until the redness and swelling disappear." b. "I have been putting ice on my son's arm to relieve the swelling." c. "I should call the doctor if the redness disappears." d. "I have been putting a warm soak on my son's arm every 4 hours."

ANS: D Feedback A The parent should not discontinue antibiotics when signs of infection disappear. To ensure complete healing, the parent should understand that the entire course of antibiotics should be given as prescribed. B A warm soak is indicated for the treatment of cellulitis. Ice will decrease circulation to the affected area and inhibit the healing process. C The disappearance of redness indicates healing and is not a reason to seek medical advice. D Warm soaks applied every 4 hours while the child is awake increase circulation to the infected area, relieve pain, and promote healing.

What is helpful to tell a mother who is concerned about preventing sleep problems in her 2-year-old child? a. Have the child always sleep in a quiet, darkened room. b. Provide high-carbohydrate snacks before bedtime. c. Communicate with the child's daytime caretaker about eliminating the afternoon nap. d. Use a nightlight in the child's room.

ANS: D Feedback A A dark, quiet room may be scary to a preschooler. B High-carbohydrate snacks increase energy and do not promote relaxation. C Most 2-year-olds take one nap each day. Many give up the habit by age 3 years. Insufficient rest during the day can lead to irritability and difficulty sleeping at night. D The preschooler has a great imagination. Sounds and shadows can have a negative effect on sleeping behavior. Nightlights provide the child with the ability to visualize the environment and decrease the fear felt in a dark room.

Which assessment finding in a preschooler suggests the need for further investigation? a. The child is able to dress independently. b. The child rides a tricycle. c. The child has an imaginary friend. d. The child has a 2-lb weight gain in 12 months.

ANS: D Feedback A A preschool child should be able to dress independently. B A preschool child should be able to ride a tricycle. C Imaginary friends are common for preschoolers. D Preschool children gain an average of 5 pounds a year. A gain of only 2 pounds is less than half of the expected weight gain and should be investigated

A school nurse is conducting a class on safety for a group of school-age children. Which statement indicates that the children may need further teaching? a. "My sister and I know two different ways to get out of the house." b. "I can dial 911 if there is a fire or a burglar in the house." c. "My mother has told us that if we have a fire, we have to meet at the neighbor's house." d. "If there is a fire I will have to go back in for my cat Fluffy because she will be scared."

ANS: D Feedback A All children should know two different escape routes from the house, in case one is blocked. B It is important for children to be taught how to call 911 in an emergency. C All families should have a predetermined meeting place away from the house. D Children should be taught never to return to a burning house, not even for a pet.

Which is the priority concern in developing a teaching plan for the parents of a 15-month-old child? a. Toilet training guidelines b. Guidelines for weaning children from bottles c. Instructions on preschool readiness d. Instructions on a home safety assessment

ANS: D Feedback A Although it is appropriate to give parents of a 15-month-old child toilet training guidelines, the child is not usually ready for toilet training, so it is not the priority teaching intervention. B Parents of a 15-month-old child should have been advised to beginning weaning from the breast or bottle at 6 to 12 months of age. C Educating a parent about preschool readiness is important and can occur later in the parents' educational process. The priority teaching intervention for the parents of a 15-month-old child is the importance of a safe environment. D Accidents are the major cause of death in children, including deaths caused by ingestion of poisonous materials. Home and environmental safety assessments are priorities in this age-group because of toddlers' increased motor skills and independence, which puts them at greater risk in an unsafe environment.

The mother of a 9-month-old infant is concerned because the infant cries when approached by an unknown shopper at the grocery store. What is the best response for the nurse to make to the mother? a. "You could consider leaving the infant more often with other people so he can adjust." b. "You might consider taking him to the doctor because he may be ill." c. "Have you noticed whether the baby is teething?" d. "This is a sign of stranger anxiety and demonstrates healthy attachment."

ANS: D Feedback A An infant who manifests stranger anxiety is showing a normal sign of healthy attachment. This behavior peaks at 7 to 9 months and is developmentally appropriate. The mother leaving the child more often will not change this developmental response to new strangers. B Assessing developmental needs is appropriate before taking an infant to a physician. C Pain from teething expressed by the infant's cries would not occur only when the mother left the room. D The nurse can reassure parents that healthy attachment is manifested by stranger anxiety in late infancy.

A mother asks the nurse, "When should I begin to clean my baby's teeth?" What is the best response for the nurse to make? a. "You can begin when all her baby teeth are in." b. "You can easily begin now. Just put some toothpaste on a gauze pad to clean the teeth." c. "I don't think you have to worry about that until she can handle a toothbrush." d. "You can begin as soon as your child has a tooth. The easiest way is to take cotton swabs or a face cloth and just wipe the teeth. Toothpaste is not necessary."

ANS: D Feedback A An infant's teeth need to be cleaned as soon as they erupt. Waiting until all the baby teeth are in is inappropriate and prolongs cleaning until 2 years of age. B Because toothpaste contains fluoride and infants will swallow the toothpaste, parents should avoid its use. C The infant's teeth need to be cleaned by the parent as soon as they erupt. Even when a child has the ability to hold a toothbrush, the parent should continue cleaning the child's teeth. D An infant's teeth need to be cleaned as soon as they erupt. Cleaning the teeth with cotton swabs or a face cloth is appropriate.

What is the best response a nurse can make to a 15-year-old girl who has verbalized a desire to have a baby? a. "Have you talked with your parents about this?" b. "Do you have plans to continue school?" c. "Will you be able to support the baby?" d. "Can you tell me how your life will be if you have an infant?"

ANS: D Feedback A Asking the teenager whether she has talked to her parents is not particularly helpful to the teen or the nurse and may terminate the communication. B A direct question about continuing school will not facilitate communication. Open-ended questions encourage communication. C Asking the teenager about how she will support the child will not facilitate communication. Open-ended questions encourage communication. D Having the teenager describe how the infant will affect her life will allow the teen to think more realistically. Her description will allow the nurse to assess the teen's perception and reality orientation.

Which statement best describes why infants are at greater risk for dehydration than older children? a. Infants have an increased ability to concentrate urine. b. Infants have a greater volume of intracellular fluid. c. Infants have a smaller body surface area. d. Infants have an increased extracellular fluid volume.

ANS: D Feedback A Because the kidneys are immature in early infancy, there is a decreased ability to concentrate the urine. B Infants have a larger proportion of fluid in the extracellular space. C Infants have proportionately greater body surface area in relation to body mass, which creates the potential for greater fluid loss through the skin and gastrointestinal tract. D The larger ratio of extracellular fluid to intracellular fluid predisposes the infant to dehydration.

The mother of a 10-month-old infant asks the nurse about beginning to wean her child from his bottle. Which statement by the mother suggests that the child is not ready to be weaned? a. "My son is frequently throwing his bottle down." b. "The baby takes a few ounces of formula from the bottle." c. "He is constantly chewing on the nipple. It concerns me." d. "He consistently is sucking."

ANS: D Feedback A Decreased interest in the bottle starts between 6 and 12 months. Throwing the bottle down is a sign of a decreased interest in the bottle. B When the child is taking more fluids from a cup and decreasing amounts from the bottle, the child is demonstrating a readiness for weaning. C Chewing on the nipple is another sign that the infant is ready to be weaned. D Consistent sucking is a sign that the child is not ready to be weaned.

A nurse is teaching parents about diarrhea. Which statement by the parents indicates understanding of the teaching? a. Diarrhea results from a fluid deficit in the small intestine. b. Organisms destroy intestinal mucosal cells, resulting in an increased intestinal surface area. c. Malabsorption results in metabolic alkalosis. d. Increased motility results in impaired absorption of fluid and nutrients.

ANS: D Feedback A Diarrhea results from fluid excess in the small intestine. B Destroyed intestinal mucosal cells result in decreased intestinal surface area. C Loss of electrolytes in the stool from diarrhea results in metabolic acidosis. D Increased motility and rapid emptying of the intestines result in impaired absorption of nutrients and water. Electrolytes are drawn from the extracellular space into stool, and dehydration results.

What do parents of preschool children need to understand about discipline? a. Both parents and the child should agree on the method of discipline. b. Discipline should involve some physical restriction. c. The method of discipline should be consistent with the discipline methods of the child's peers. d. Discipline should include positive reinforcement of desired behaviors.

ANS: D Feedback A Discipline does not need to be agreed on by the child. Preschoolers feel secure with limits and appropriate, consistent discipline. Both parents should be in agreement so that the discipline is consistently applied. B Discipline does not necessarily need to include physical restriction. C Discipline does not need to be consistent with that of the child's peers. D Effective discipline strategies should involve a comprehensive approach that includes consideration of the parent-child relationship, reinforcement of desired behaviors, and consequences for negative behaviors.

The most common cause of death in the adolescent age-group involves a. Drownings b. Firearms c. Drug overdoses d. Motor vehicles

ANS: D Feedback A Drownings are major concerns in adolescence but do not cause the majority of deaths. B Firearms are major concerns in adolescence but do not cause the majority of deaths. C Drug overdoses are major concerns in adolescence but do not cause the majority of deaths. D Risk taking behaviors play a major role in the high incidence of motor vehicle injuries and death among teenagers i.e. alcohol use, failure to wear a seatbelt, and inexperience.

The school nurse has been asked to begin teaching sex education in the 5th grade. The nurse should recognize that a. Children in 5th grade are too young for sex education. b. Children should be discouraged from asking too many questions. c. Correct terminology should be reserved for children who are older. d. Sex can be presented as a normal part of growth and development.

ANS: D Feedback A Fifth graders are usually 10 to 11 years old. This age is not too young to speak about physiologic changes in their bodies. B They should be encouraged to ask questions. C Preadolescents need precise and concrete information. D When sexual information is presented to school-age children, sex should be treated as a normal part of growth and development.

What is the priority nursing intervention for a 6-month-old infant hospitalized with diarrhea and dehydration? a. Estimating insensible fluid loss b. Collecting urine for culture and sensitivity c. Palpating the posterior fontanel d. Measuring the infant's weight

ANS: D Feedback A Infants have a greater total body surface area and therefore a greater potential for fluid loss through the skin. It is not possible to measure insensible fluid loss. B Urine for culture and sensitivity is not usually part of the treatment plan for the infant who is dehydrated from diarrhea. C The posterior fontanel closes by 2 months of age. The anterior fontanel can be palpated during an assessment of an infant with dehydration. D Weight is a crucial indicator of fluid status. It is an important criterion for assessing hydration status and response to fluid replacement.

The nurse is discussing with a parent group the importance of fluoride for healthy teeth. What should the nurse recommend? a. Use fluoridated mouth rinses in children older than 1 year. b. Brush teeth with fluoridated toothpaste unless fluoride content of water supply is adequate. c. Give fluoride supplements to breastfed infants beginning at age 1 month. d. Determine whether water supply is fluoridated.

ANS: D Feedback A It is difficult to teach this age-group to spit out the mouthwash. Swallowing fluoridated mouthwashes can contribute to fluorosis. B Fluoridated toothpaste is still indicated, but very small amounts are used. C Fluoride supplementation is not recommended until after age 6 months. D The decision about fluoride supplementation cannot be made until it is known whether the water supply contains fluoride and the amount.

Which comment is most developmentally typical of a 7-year-old boy? a. "I am a Power Ranger, so don't make me angry." b. "I don't know whether I like Mary or Joan better." c. "My mom is my favorite person in the world." d. "Jimmy is my best friend."

ANS: D Feedback A Magical thinking is developmentally appropriate for the preschooler. B Opposite-sex friendships are not typical for the 7-year-old child. C Seven-year-old children socialize with their peers, not their parents. D School-age children form friendships with peers of the same sex, those who live nearby, and other children who have toys that they enjoy.

A parent asks the nurse about negativism in toddlers. The most appropriate recommendation is to a. Punish the child b. Provide more attention c. Ask the child to not always say "no" d. Reduce the opportunities for a "no" answer

ANS: D Feedback A Negativism is not an indication of stubbornness or insolence and should not be punished. B The negativism is not a function of attention; the child is testing limits to gain an understanding of the world. C The toddler is too young for this approach. D The nurse should suggest to the parent that questions should be phrased with realistic choices rather than yes or no answers. This provides a sense of control for the toddler and reduces the opportunity for negativism.

Which behavior is not demonstrated in the 8-year-old child? a. Understands that his or her point of view is not the only one b. Enjoys telling riddles and silly jokes c. Understands that pouring liquid from a small to large container does not change the amount d. Engages in fantasy and magical thinking

ANS: D Feedback A School-age children enter the stage of concrete operations. They learn that their point of view is not the only one. B The school-age child has a sense of humor. The child's increased language mastery and increased logic allow for appreciation of plays on words, jokes, and incongruities. C The school-age child understands that properties of objects do not change when their order, form, or appearance does. D The preschool-age child engages in fantasy and magical thinking. The school-age child moves away from this type of thinking and becomes more skeptical and logical. Belief in Santa Claus or the Easter Bunny ends in this period of development.

Which statement made by a mother of a school-age boy indicates a need for further teaching? a. "My child is playing soccer this year." b. "He is always busy with his friends playing games. He is very active." c. "I limit his television watching to about 2 hours a day." d. "I am glad his coach is a good role model. He emphasizes the importance of winning in today's society. The kids really are disciplined."

ANS: D Feedback A Team sports such as soccer are appropriate for exercise and refinement of motor skills. B School-age children need to participate in physical activities, which contribute to their physical fitness skills and well-being. C Limiting television to 2 hours a day is an appropriate restriction. School-age children should be encouraged to participate in physical activities. D Team sports are important for the development of sportsmanship and teamwork and for exercise and refinement of motor skills. A coach who emphasizes winning and strict discipline is not appropriate for children in this age-group.

When planning care for adolescents, the nurse should: a. Teach parents first, and they, in turn, will teach the teenager. b. Provide information for their long-term health needs because teenagers respond best to long-range planning. c. Maintain the parents' role by providing explanations for treatment and procedures to the parents only. d. Give information privately to adolescents about how they can manage the specific problems that they identify.

ANS: D Feedback A Teenagers are socially and cognitively at the developmental stage where the health care provider can teach them. B Teenagers are more interested in immediate health care needs than in long-term needs. C Teenagers are at the developmental level that allows them to receive explanations about health care directly from the nurse. D Problems that teenagers identify and are interested in are typically the problems that they are the most willing to address. Confidentiality is important to adolescents. Adolescents prefer to confer privately (without parents) with the nurse and health care provider.

In providing anticipatory guidance to parents, which parental behavior is the most important in fostering moral development? a. Telling the child what is right and wrong b. Vigilantly monitoring the child and her peers c. Weekly family meetings to discuss behavior d. Living as the parents say they believe

ANS: D Feedback A Telling the child what is right and wrong is not effective unless the child has experienced what she hears. Parents need to live according to the values they are teaching to their children. B Vigilant monitoring of the child and her peers is an inappropriate action for the parent to initiate. It does not foster moral development and reasoning in the child. C Weekly family meetings to discuss behaviors may or may not be helpful in the development of moral reasoning. D Parents living what they believe gives nonambivalent messages and fosters the child's moral development and reasoning.

Which statement concerning physiologic factors is true? a. The infant has a slower metabolic rate than an adult. b. An infant has an inability to digest protein and lactase. c. Infants have a slower circulatory response than adults do. d. The kidneys of an infant are less efficient in concentrating urine than an adult's kidneys.

ANS: D Feedback A The infant's metabolic rate is faster, not slower, than an adult's. B Although the newborn infant's gastrointestinal system is immature, it is capable of digesting protein and lactase, but the ability to digest and absorb fat does not reach adult levels until approximately 6 to 9 months of age. C Circulation is faster in infants than in adults. D The infant's kidneys are not as effective at concentrating urine compared with an adult's because of immaturity of the renal system and slower glomerular filtration rates. This puts the infant at greater risk for fluid and electrolyte imbalance.

A child has a 2-day history of vomiting and diarrhea. He has hypoactive bowel sounds and an irregular pulse. Electrolyte values are sodium, 139 mEq/L; potassium, 3.3 mEq/L; and calcium, 9.5 mg/dL. This child is likely to have which of the following electrolyte imbalances? a. Hyponatremia b. Hypocalcemia c. Hyperkalemia d. Hypokalemia

ANS: D Feedback A The normal serum sodium level is 135 to 145 mEq/L. A level of 139 mEq/L is within normal limits. B A serum calcium level less than 8.5 mg/dL is considered hypocalcemia. C A serum potassium level greater than 5 mEq/L is considered hyperkalemia. D A serum potassium level less than 3.5 mEq/L is considered hypokalemia. Clinical manifestations of hypokalemia include muscle weakness, decreased bowel sounds, cardiac irregularities, hypotension, and fatigue.

The parents of a newborn say that their toddler "hates the baby...he suggested that we put him in the trash can so the trash truck could take him away." The nurse's best reply is a. "Let's see if we can figure out why he hates the new baby." b. "That's a strong statement to come from such a small boy." c. "Let's refer him to counseling to work this hatred out. It's not a normal response." d. "That is a normal response to the birth of a sibling. Let's look at ways to deal with this."

ANS: D Feedback A The toddler does not hate the infant. This is an expected response to the changes in routines and attention that affect the toddler. B This is a normal response. The toddler can be provided with a doll to care for and tend to the doll's needs at the same time the parent is performing similar care for the newborn. C The toddler does not hate the infant. This is an expected response to the changes in routines and attention that affect the toddler. D The arrival of a new infant represents a crisis for even the best-prepared toddler. Toddlers have their entire schedule and routines disrupted because of the new family member. The nurse should work with parents on ways to involve the toddler in the newborn's care and to help focus attention on the toddler.

In girls, the initial indication of puberty is a. Menarche b. Growth spurt c. Growth of pubic hair d. Breast development

ANS: D Feedback A The usual sequence of secondary sexual characteristic development in girls is breast changes, rapid increase in height and weight, growth of pubic hair, appearance of axillary hair, menstruation, and abrupt deceleration of linear growth. B The usual sequence of secondary sexual characteristic development in girls is breast changes, rapid increase in height and weight, growth of pubic hair, appearance of axillary hair, menstruation, and abrupt deceleration of linear growth. C The usual sequence of secondary sexual characteristic development in girls is breast changes, rapid increase in height and weight, growth of pubic hair, appearance of axillary hair, menstruation, and abrupt deceleration of linear growth. D In most girls, the initial indication of puberty is the appearance of breast buds, an event known as thelarche.

A father tells the nurse that his daughter wants the same plate and cup used at every meal, even if they go to a restaurant. The nurse should explain that this is a. A sign the child is spoiled b. A way to exert unhealthy control c. Regression, common at this age d. Ritualism, common at this age

ANS: D Feedback A This is not indicative of a child who has unreasonable expectations, but rather normal development. B Toddlers use ritualistic behaviors to maintain necessary structure in their lives. C This is not regression, which is a retreat from a present pattern of functioning. D The child is exhibiting the ritualism that is characteristic at this age. Ritualism is the need to maintain the sameness and reliability. It provides a sense of comfort to the toddler. It will dictate certain principles in feeding practices, including rejecting a favorite food because it is served in a different container.

A 17-month-old child is expected to be in what stage according to Piaget? a. Trust b. Preoperations c. Secondary circular reaction d. Sensorimotor period

ANS: D Feedback A Trust is Erikson's first stage. B Preoperations is the stage of cognitive development usually present in older toddlers and preschoolers. C Secondary circular reactions last from approximately ages 4 to 8 months. D The 17-month-old is in the fifth stage of the sensorimotor phase, tertiary circular reactions. Learning in this stage occurs mainly by trial and error.

You are preparing immunizations for a 12-month-old child who is immunocompromised. Which immunizations cannot be given? Select all that apply. a. DTaP b. HepA c. IPV d. Varicella e. MMR

ANS: D, E Feedback Correct Children who are immunologically compromised should not receive live viral vaccines. Varicella is a live vaccine and should not be given except in special circumstances. MMR is a live vaccine and should not be given to immunologically compromised children. Incorrect DTaP, HepA, and IPV can be given safely.

The number of hours spent sleeping decreases as the child grows older. Children ages 6 and 7 years require approximately 9 or 10 hours of sleep per night. Is this statement true or false?

ANS: F Children ages 6 and 7 actually need approximately 12 hours of sleep per night. Some children also continue to need an afternoon nap or quiet time to restore energy levels. The 12-year-old needs approximately 9 to 10 hours of sleep at night. Adequate sleep is important for school performance and physical growth. Inadequate sleep can cause irritability, inability to concentrate, and poor school performance.

The nurse who provides care for young children with fluid and electrolyte imbalance understands that they are more vulnerable to changes in fluid balance than adults. Under normal conditions the amount of fluid ingested during the day should equal the amount of fluid lost. Sensible water loss is that which occurs through the respiratory tract and skin. Is this statement true or false?

ANS: F Sensible water loss occurs through urine output. Insensible water loss occurs through the skin and respiratory tract. Insensible water loss per unit of body weight is significantly higher in infants and young children due to the faster respiratory rate and higher evaporative water losses.

The rate of Sudden Infant Death Syndrome (SIDS), now the third leading cause of death in infants, has increased despite international efforts and the Back to Sleep campaign. Is this statement true or false?

ANS: F This statement is incorrect. SIDS, which for a long time was the second leading cause of infant deaths, has decreased in part because of the Back to Sleep program. It is important for both hospital and clinic nurses to educate parents on safe sleep strategies for their infant.

Electric injury to a child often results in instant death because the electric current disrupts the rhythm of the heart. Is this statement true or false?

ANS: T The child who does not die instantly after an electrical injury is at risk for cardiac arrest or dysrhythmia, tissue damage, myoglobinuria, and metabolic acidosis.

Breastfeeding is the ideal method for providing nutrition to the human infant and is recommended by the American Heart Association, the American Academy of Pediatrics, and the World Health Organization. Infants should be exclusively breastfed for a minimum of 4 months and preferably 6 months. Is this statement true or false?

ANS: T This statement is correct. Solid food should not be introduced until 4 to 6 months of age. Breastfeeding should accompany solid food introduction until a minimum of 1 year of age (according to the AAP) and a minimum of 2 years of age (according to the WHO).

Alterations in acid-base balance can affect cellular metabolism and enzymatic processes. When alterations in pH become too much for buffer systems to handle, compensatory mechanisms are activated. If the pH drops below normal than acidosis will occur. Is this statement true or false?

ANS: T Acidosis is the result of a drop in blood pH. The respiratory rate and depth will increase, removing carbon dioxide and raising blood pH. Conversely in the presence of alkalosis, respiratory rate and depth decrease, lowering blood pH.

Parents are often concerned about their toddler's interest in and curiosity about gender differences. Sex play and masturbation are common among toddlers. Is this statement true or false?

ANS: T Nurses can reassure parents that self-exploration and exploration of another toddler's body is normal behavior during early childhood. Parents should respect the child's curiosity as normal and not judge them as being "bad."

The use of electronic or digital media for communication has had a negative effect on the language development of adolescents. Is this statement true or false?

ANS: T Text messaging, instant messaging, blogs, and Twitter all contribute to abbreviated communication techniques, which eliminate not only grammar and sentence construction, but also word development (e.g., using ur, for you are).

A 4-year-old has had diarrhea for several days, and her perineum is inflamed and almost excoriated. What nursing actions are indicated? Select all that apply. A. Gently wash the perineum with cold water and mild soap after each stool. B. Apply an ointment to the inflamed area to provide a moisture barrier. C. Place the child without underwear for brief periods to allow air to the area. D. Turn the child at least every 2 hours.

B, C, D Applying an ointment to the inflamed area to provide a moisture barrier is important. Placing the child without underwear for brief periods to allow air to the area often helps heal the area. Turning the child at least every 2 hours keeps pressure off the skin and facilitates circulation to the affected area. Gently wash the perineum with warm water and mild soap after each stool.

A preschooler with vomiting and diarrhea lost 0.5 kg of weight since being weighed in the pediatrician's office prior to admission to the hospital. How much fluid would the nurse calculate that this child has lost? A. 250 mL B. 500 mL C. 750 mL D. 1000 mL

B. 500 mL One milliliter of body fluid is approximately equal to 1 g of body weight, so a weight loss or gain of 1 kg represents 1 L, or 1000 mL. A half-kilogram loss would be 500 mL.

A preschooler with severe vomiting and diarrhea was admitted to the hospital. The vomiting has stopped, and rehydration was begun intravenously. When should the nurse begin feeding the child solid food? A. When the parents give their permission to feed their child B. After the child has been rehydrated C. After the diarrhea has stopped for 24 hours D. When the IV rehydration can be stopped

B. After the child has been rehydrated Feeding of solids or formula is started as soon as the child is rehydrated. Children should be encouraged to eat frequently—every 3 to 4 hours. Parents should be instructed that although stool output may increase, feeding will not prolong diarrhea, and the child will be absorbing necessary nutrients and calories. Parents should be instructed that although stool output may increase, feeding will not prolong diarrhea, and the child will be absorbing necessary nutrients and calories. The intravenous solutions may run a little longer to ensure that the child remains hydrated. It is not up to the parents to decide when resumption of solid food begins.

A toddler is hospitalized with severe dehydration. The nurse should assess the child for which possible complication? A. Hypertension B. Hypokalemia C. A rapid, bounding pulse D. Decreased specific gravity

B. Hypokalemia Hypokalemia is a concern in severe dehydration. A rapid, thready pulse would be seen in severe dehydration. The urine would be concentrated, so the specific gravity would increase. The child needs to be monitored for hypotension.

The nurse is teaching a parents' class about when to call the pediatrician's office if vomiting and diarrhea in their toddlers. Instruction by the nurse is correct if the nurse includes which information? Select all that apply. A. If their child doesn't urinate for longer than 4 hours B. If their child's fontanel appears sunken C. If crying produces no tears D. When the diarrhea has been present for 24 hours E. If the toddler has a fever (>39° C [102° F]) F. If severe abdominal cramps occur

C, D, E, F If crying produces no tears, the pediatrician should be notified. When the diarrhea has been present for 24 hours, the pediatrician should be notified. If the toddler has a fever >39° C (102° F), the pediatrician should be notified. If severe abdominal cramps occur, the pediatrician should be notified. If their toddler doesn't urinate for longer than 6 hours, the pediatrician should be notified. The fontanels disappear by 18 months of age.

A school-age child with acute diarrhea and mild dehydration is being given oral rehydration solutions (ORS). The child's mother calls the clinic nurse to report the child has occasional vomiting. What is the appropriate recommendation by the nurse? A. Bring the child to the hospital for intravenous fluids. B. Alternate giving the child ORS and carbonated drinks. C. Continue to give the child ORS frequently in small amounts. D. Maintain the child on NPO for 8 hours and resume ORS if vomiting subsides.

C. Continue to give the child ORS frequently in small amounts. Vomiting is not a contraindication to the use of ORS unless it is severe. The mother should continue to give the ORS in small amounts and at frequent intervals. NPO status is not indicated. Frequent intake of ORS in small amounts is recommended. A school-age child with mild dehydration can be rehydrated safely at home with oral solutions. Carbonated drinks should not be given to the child. They may have a high carbohydrate content and contain caffeine, which is a diuretic.

A 2-month-old infant has been brought to the emergency department because of diarrhea and vomiting for the past 48 hours. Why should the pediatric nurse expect the infant to be at a greater risk for fluid and electrolyte imbalances than older children? A. Infants have a lower metabolic rate than older children. B. Infants have a decreased surface area. C. Immature renal function is common in infants. D. The infants' daily exchange of extracellular fluid is decreased.

C. Immature renal function is common in infants. Infants' kidneys are unable to concentrate or dilute urine, conserve or excrete sodium, and acidify urine, and their bodies have a higher percentage of fluid per weight than older children. There is an increased amount of extracellular fluid in the infant. Forty percent of a neonate's body fluid is extracellular fluid, compared with 20% in an adult. Fluid is lost from the extracellular space first. Infants have a higher metabolic rate. Infants have a proportionately greater body surface area, which allows for greater insensible water loss.

The nurse has measured the urinary output for the 12-hour shift and has 340 mL for a 12-kg toddler. What is the normal range of urinary output for this child for a 12-hour shift so the nurse can evaluate the output obtained? A. 238 to 366 mL B. 246 to 398 mL C. 274 to 416 mL D. 288 to 432 mL

D. 288 to 432 mL

A nurse is evaluating an infant brought to the clinic with severe diarrhea. What signs and symptoms should the nurses identify as indicating the infant has severe dehydration? A. Tachycardia, decreased tears, 5% weight loss and skin tenting B. Normal pulse rate, decreased blood pressure, intense thirst, and increased crying C. Irritability, moderate thirst, a flat fontanel, and sucking on his hands D. Tachycardia, capillary refill greater than 3 seconds, and sunken eyes and fontanel

D. Tachycardia, capillary refill greater than 3 seconds, and sunken eyes and fontanel Tachycardia, capillary refill greater than 3 seconds, and sunken eyes and fontanel are the symptoms of severe dehydration. In severe dehydration, tachycardia, decreased tears, a 15% weight loss, and skin tenting are present. Tachycardia, orthostatic hypotension and shock, and intense thirst would be expected. Crying may or not be present or increased due to lack of energy. The infant would be extremely irritable, with sunken eyes and fontanel.


Kaugnay na mga set ng pag-aaral

BIOLÓGIA 1. / DIFFÚZIÓ, OZMÓZIS /

View Set

AP Comparative Government: Iran Vocabulary

View Set

Servsafe Manager Test Chapters 1-5

View Set

California Real Estate Law Fair Housing Laws 1

View Set

Med Surg - Exam 3 (ATI, EOC, NCLEX)

View Set

Art appreciation chapter 6 and 7

View Set